clinical scenarios in surgery - booksca.ca...clinical scenarios in surgery decision making and...

45
Clinical Scenarios in Surgery DECISION MAKING AND OPERATIVE TECHNIQUE Editors Justin B. Dimick, MD, MPH Assistant Professor of Surgery Chief, Division of Minimally Invasive Surgery Department of Surgery University of Michigan Ann Arbor, Michigan Gilbert R. Upchurch Jr., MD William H. Muller, Jr. Professor Chief of Vascular and Endovascular Surgery University of Virginia Charlottesville, Virginia Christopher J. Sonnenday, MD, MHS Assistant Professor of Surgery Assistant Professor of Health Management & Policy University of Michigan Ann Arbor, Michigan

Upload: others

Post on 21-Jul-2021

13 views

Category:

Documents


0 download

TRANSCRIPT

Page 1: Clinical Scenarios in Surgery - Booksca.ca...Clinical Scenarios in Surgery DECISION MAKING AND OPERATIVE TECHNIQUE Editors Justin B. Dimick, MD, MPH Assistant Professor of Surgery

Clinical Scenarios in SurgeryDECISION MAKING AND OPERATIVE TECHNIQUE

Editors

Justin B. Dimick, MD, MPHAssistant Professor of SurgeryChief, Division of Minimally Invasive SurgeryDepartment of SurgeryUniversity of MichiganAnn Arbor, Michigan

Gilbert R. Upchurch Jr., MDWilliam H. Muller, Jr. ProfessorChief of Vascular and Endovascular SurgeryUniversity of VirginiaCharlottesville, Virginia

Christopher J. Sonnenday, MD, MHSAssistant Professor of SurgeryAssistant Professor of Health Management & PolicyUniversity of MichiganAnn Arbor, Michigan

Dimick_FM.indd iDimick_FM.indd i 3/30/2012 4:47:13 PM3/30/2012 4:47:13 PM

Page 2: Clinical Scenarios in Surgery - Booksca.ca...Clinical Scenarios in Surgery DECISION MAKING AND OPERATIVE TECHNIQUE Editors Justin B. Dimick, MD, MPH Assistant Professor of Surgery

Acquisitions Editor: Brian BrownProduct Manager: Brendan HuffmanProduction Manager: Bridgett DoughertySenior Manufacturing Manager: Benjamin RiveraMarketing Manager: Lisa LawrenceDesign Coordinator: Joan WendtProduction Service: SPi Global

Copyright © 2012 by LIPPINCOTT WILLIAMS & WILKINS, a WOLTERS KLUWER businessTwo Commerce Square2001 Market StreetPhiladelphia, PA 19103 USALWW.com

All rights reserved. This book is protected by copyright. No part of this book may be reproduced in any form by any means, including photocopying, or utilized by any information storage and retrieval system without written permission from the copyright owner, except for brief quotations embodied in critical articles and reviews. Materials appearing in this book prepared by individuals as part of their offi cial duties as U.S. government employees are not covered by the above-mentioned copyright.

Printed in China

Library of Congress Cataloging-in-Publication DataClinical scenarios in surgery : decision making and operative technique / [edited by] Justin B. Dimick. — 1st ed. p. ; cm. Includes bibliographical references and index. ISBN 978-1-60913-972-8 I. Dimick, Justin B. [DNLM: 1. Surgical Procedures, Operative—methods—Case Reports. WO 16] 617—dc23 2012007290

Care has been taken to confi rm the accuracy of the information presented and to describe generally accepted practices. However, the authors, editors, and publisher are not responsible for errors or omissions or for any consequences from application of the information in this book and make no warranty, expressed or implied, with respect to the currency, completeness, or accuracy of the con-tents of the publication. Application of the information in a particular situation remains the profes-sional responsibility of the practitioner.

The authors, editors, and publisher have exerted every effort to ensure that drug selection and dosage set forth in this text are in accordance with current recommendations and practice at the time of publication. However, in view of ongoing research, changes in government regulations, and the constant fl ow of information relating to drug therapy and drug reactions, the reader is urged to check the package insert for each drug for any change in indications and dosage and for added warnings and precautions. This is particularly important when the recommended agent is a new or infrequently employed drug.

Some drugs and medical devices presented in the publication have Food and Drug Administration (FDA) clearance for limited use in restricted research settings. It is the responsibility of the health care provider to ascertain the FDA status of each drug or device planned for use in their clinical practice.

To purchase additional copies of this book, call our customer service department at (800) 638-3030 or fax orders to (301) 223-2320. International customers should call (301) 223-2300.

Visit Lippincott Williams & Wilkins on the Internet: at LWW.com. Lippincott Williams & Wilkins customer service representatives are available from 8:30 am to 6 pm, EST.

10 9 8 7 6 5 4 3 2 1

Dimick_FM.indd iiDimick_FM.indd ii 3/30/2012 4:47:13 PM3/30/2012 4:47:13 PM

Page 3: Clinical Scenarios in Surgery - Booksca.ca...Clinical Scenarios in Surgery DECISION MAKING AND OPERATIVE TECHNIQUE Editors Justin B. Dimick, MD, MPH Assistant Professor of Surgery

To my wife, Anastasia, and our children, Mary and Paul.

—Justin B. Dimick

To my wife Nancy, and my boys, Rivers, Walker, Joe, and Antione. Thanks for moving with me to Virginia!

—Gilbert R. Upchurch Jr.

To the general surgery residents of the University of Michigan, for their constant inspiration and dedication to patient care.

—Christopher J. Sonnenday

Dimick_FM.indd iiiDimick_FM.indd iii 3/30/2012 4:47:13 PM3/30/2012 4:47:13 PM

Page 4: Clinical Scenarios in Surgery - Booksca.ca...Clinical Scenarios in Surgery DECISION MAKING AND OPERATIVE TECHNIQUE Editors Justin B. Dimick, MD, MPH Assistant Professor of Surgery

Dimick_FM.indd ivDimick_FM.indd iv 3/30/2012 4:47:13 PM3/30/2012 4:47:13 PM

This page intentionally left blank.

Page 5: Clinical Scenarios in Surgery - Booksca.ca...Clinical Scenarios in Surgery DECISION MAKING AND OPERATIVE TECHNIQUE Editors Justin B. Dimick, MD, MPH Assistant Professor of Surgery

v

Contributors

Edouard Aboian, MDClinical Fellow in Vascular SurgeryDepartment of Vascular SurgeryMaimonides Medical CenterBrooklyn, New York

Lifestyle-Limiting Claudication

Daniel Albo, MD, PhDChief, Division of Surgical OncologyDepartment of SurgeryBaylor College of MedicineDirector, GI Oncology ProgramDan L. Duncan Cancer CenterBaylor College of MedicineHouston, Texas

Splenic Flexure Colon Cancer

Amy K. Alderman, MD, MPHThe Swan CenterAlpharetta, Georgia

Breast Reconstruction

Steven R. Allen, MDAssistant Professor of SurgeryDepartment of Traumatology, Surgical Critical Care

and Emergency SurgeryUniversity of PennsylvaniaPhiladelphia, Pennsylvania

Adrenal Insuffi ciency

John B. Ammori, MDAssistant ProfessorDepartment of SurgeryDivision of General and Oncologic SurgeryCase Western Reserve UniversityAttending SurgeonDepartment of SurgeryDivision of General and Oncologic SurgeryUniversity Hospitals Case Medical CenterCleveland, Ohio

Gastrointestinal Stromal Tumor

Christopher D. Anderson, MDAssociate Professor of SurgeryChief, Division of Transplant and Hepatobiliary

SurgeryDepartment of SurgeryUniversity of Mississippi Medical CenterJackson, Mississippi

Acute Liver Failure

Stanley W. Ashley, MDFrank Sawyer Professor of SurgeryHarvard Medical SchoolChief Medical Offi cerSenior Vice President for Medical AffairsAdministrationBrigham and Women’s HospitalBoston, Massachusetts

Severe Acute Pancreatitis

Samir S. Awad, MDAssociate Professor of SurgeryChief, Section of Surgical Critical CareProgram Director Surgical Critical CareDepartment of SurgeryBaylor College of MedicineAssociate Chief of SurgeryMedical Director SICUMichael E. DeBakey VAMCHouston, Texas

Fulminant Clostridium Diffi cile Colitis

Douglas C. Barnhart, MD, MSPHAssociate ProfessorDivision of Pediatric SurgeryUniversity of UtahAttending SurgeonPediatric SurgeryPrimary Children’s Medical CenterSalt Lake City, Utah

Palpable Abdominal Mass in a Toddler

Dimick_FM.indd vDimick_FM.indd v 3/30/2012 4:47:13 PM3/30/2012 4:47:13 PM

Page 6: Clinical Scenarios in Surgery - Booksca.ca...Clinical Scenarios in Surgery DECISION MAKING AND OPERATIVE TECHNIQUE Editors Justin B. Dimick, MD, MPH Assistant Professor of Surgery

vi Contributors

Brendan J. Boland, MDDepartment of SurgeryLAC+USC Medical CenterLos Angeles, California

Variceal Bleeding and Portal Hypertension

Melissa Boltz, DO, MBAResident PhysicianDepartment of SurgeryPenn State College of MedicinePenn State Milton S. Hershey Medical CenterHershey, Pennsylvania

Incidental Adrenal Mass

Tara M. Breslin, MDAssistant Professor of SurgeryDepartment of SurgeryUniversity of MichiganAnn Arbor, Michigan

Ductal Carcinoma In Situ

Adam S. Brinkman, MDResidentDivision of General SurgeryUniversity of Wisconsin School of Medicine and

Public HealthMadison, Wisconsin

Malrotation and Midgut Volvulus

Malcolm V. BrockAssociate Professor of SurgeryAssociate Professor of OncologyDirector of Clinical and Translational Research

in Thoracic SurgeryThe Johns Hopkins HospitalBaltimore, Maryland

Solitary Pulmonary Nodule

James T. Broome, MDAssistant Professor of SurgeryDivision of Surgical Oncology and Endocrine

SurgeryVanderbilt UniversityNashville, Tennessee

Persistent Hyperparathyroidism

William C. Beck, MDResident Physician, General SurgeryDepartment of General SurgeryVanderbilt UniversityVanderbilt University Medical CenterNashville, Tennessee

Cholangitis

Natasha S. Becker, MD, MPHFellow in Surgical Critical CareBaylor College of MedicineHouston, Texas

Fulminant Clostridium Diffi cile Colitis

Filip Bednar, MDHouse Offi cerDepartment of SurgeryUniversity of MichiganAnn Arbor, Michigan

Duodenal Injury

Jessica M. Bensenhaver, MDSurgical Breast Oncology FellowClinical Lecturer of SurgeryUniversity of Michigan Health SystemsAnn Arbor, Michigan

Ductal Carcinoma In Situ

Noelle L. Bertelson, MDLaparoscopic Colorectal Surgery FellowSurgeryMayo ClinicPhoenix, Arizona

Large Bowel Obstruction from Colon Cancer

Avi Bhavaraju, MDInstructor in SurgeryDivision of Trauma & Surgical Critical CareVanderbilt UniversityNashville, Tennessee

Pelvic Fracture

James H. Black, III, MDBertram M. Bernheim, MDAssociate Professor of SurgeryDepartment of SurgeryJohns Hopkins University School of MedicineAttending SurgeonDepartment of SurgeryJohns Hopkins HospitalBaltimore, Maryland

Acute Mesenteric Ischemia

Dimick_FM.indd viDimick_FM.indd vi 3/30/2012 4:47:13 PM3/30/2012 4:47:13 PM

Page 7: Clinical Scenarios in Surgery - Booksca.ca...Clinical Scenarios in Surgery DECISION MAKING AND OPERATIVE TECHNIQUE Editors Justin B. Dimick, MD, MPH Assistant Professor of Surgery

Contributors vii

Anthony G. Charles, MD, MPHAssistant Professor of SurgeryDirector of Adult ECMO, Associate Chief of Surgical

Critical CareDepartment of SurgeryUniversity of North CarolinaChapel Hill, North Carolina

Acute Renal Failure

Herbert Chen, MDLayton Rikkers Chair in Surgical Leadership’s

ProfessorDepartment of SurgeryUniversity of WisconsinChairman, Division of General SurgerySurgeryUniversity of Wisconsin Hospitals and ClinicsMadison, Wisconsin

Medullary Thyroid Cancer

Steven Chen, MD, MBAAssociate ProfessorDepartment of SurgeryDivision of Surgical OncologyCity of Hope National Medical CenterDuarte, California

Advanced Breast Cancer

Hueylan Chern, MDAssistant ProfessorDepartment of SurgeryUniversity of CaliforniaSan Francisco, California

Crohn’s Disease with Small Bowel Stricture

Albert Chi, MDAssistant Professor of SurgeryJohns Hopkins HospitalDivision of Acute Care SurgeryBaltimore, Maryland

Penetrating Chest Injury

Sara E. Clark, MDResidentDepartment of SurgeryUniversity of South FloridaTampa General HospitalTampa, Florida

Small Bowel Obstruction

Steven W. Bruch, MD, MScClinical Associate ProfessorDepartment of SurgeryUniversity of MichiganPediatric SurgeonDepartment of SurgeryMott Children’s HospitalAnn Arbor, Michigan

Tracheoesophageal Fistula

Terry L. Buchmiller, MDAssistant ProfessorDepartment of SurgeryHarvard Medical SchoolStaff SurgeonDepartment of SurgeryChildren’s Hospital, BostonBoston, Massachusetts

Hepatoblastoma

Richard E. Burney, MDProfessor of SurgeryDepartment of SurgeryUniversity of MichiganAttending SurgeonUniversity of Michigan HospitalsAnn Arbor, Michigan

Perianal AbscessThrombosed Hemorrhoids

Marisa Cevasco, MD, MPHClinical FellowHarvard Medical SchoolResidentDepartment of SurgeryBrigham and Women’s HospitalBoston, Massachusetts

Severe Acute Pancreatitis

Alfred E. Chang, MDProfessor of SurgeryChief, Division of Surgical OncologyDepartment of SurgeryUniversity of MichiganAnn Arbor, Michigan

Extremity Mass (Sarcoma)

Dimick_FM.indd viiDimick_FM.indd vii 3/30/2012 4:47:13 PM3/30/2012 4:47:13 PM

Page 8: Clinical Scenarios in Surgery - Booksca.ca...Clinical Scenarios in Surgery DECISION MAKING AND OPERATIVE TECHNIQUE Editors Justin B. Dimick, MD, MPH Assistant Professor of Surgery

viii Contributors

Anastasia Dimick, MDLaing & Dimick Dermatology Ann Arbor, Michigan

Nonmelanoma Skin Cancer

Justin B. Dimick, MD, MPHAssistant Professor of SurgeryChief, Division of Minimally Invasive SurgeryDepartment of SurgeryUniversity of MichiganAnn Arbor, Michigan

Acute CholecystitisIncarcerated/Strangulated Inguinal HerniaPerforated Appendicitis

Paul D. Dimusto, MDResident in SurgeryDepartment of SurgeryUniversity of MichiganAnn Arbor, Michigan

Pulsatile Abdominal MassAsymptomatic Carotid Stenosis

Gerard M. Doherty, MDChairmanDepartment of SurgeryBoston UniversityChief of SurgeryDepartment of SurgeryBoston Medical CenterBoston, Massachusetts

Papillary Thyroid Carcinoma

Bernard J. Dubray, MDResident Physician, Division of General SurgeryResearch Fellow, Section of Abdominal

TransplantationDepartment of SurgeryWashington University in Saint LouisSaint Louis, Missouri

Acute Liver Failure

Gregory Ara DumanianProfessor of SurgeryDivision of Plastic SurgeryNorthwestern Feinberg School of MedicineChief of Plastic SurgeryDivision of Plastic SurgeryNorthwestern Memorial HospitalChicago, Illinois

Infected Ventral Hernia Mesh

Robert A. Cowles, MDAssistant ProfessorDepartment of Surgery, Division of Pediatric SurgeryColumbia University College of Physicians and

SurgeonsAssistant Attending SurgeonDepartment of SurgeryMorgan Stanley Children’s Hospital and Columbia

University Medical CenterNew York, New York

Emesis in an Infant

Eric J. Culbertson, MDHouse Offi cerDepartment of SurgeryUniversity of MichiganAnn Arbor, Michigan

Enterocutaneous Fistula

Lillian G. Dawes, MDProfessor of SurgeryDepartment of SurgeryUniversity of South FloridaGeneral SurgeonJames A. Haley Veterans HospitalTampa, Florida

Small bowel obstruction

Sebastian G. De la fuente, MDSurgical Oncology FellowCollege of MedicineUniversity of South FloridaH. Lee Moffi tt Cancer Center & Research InstituteTampa, Florida

Melanoma

Ronald P. DeMatteo, MDProfessor of SurgeryVice Chair, Department of SurgeryHead, Division of General Surgical OncologyLeslie H. Blumgart Chair in SurgeryMemorial Sloan-Kettering Cancer CenterNew York, New York

Gastrointestinal Stromal Tumor

Charles S. Dietrich III, MDChief, Gynecologic Oncology SectionDepartment of Obstetrics and GynecologyTripler Army Medical CenterHonolulu, Hawaii

Gynecologic Causes of Lower Abdominal Pain

Dimick_FM.indd viiiDimick_FM.indd viii 3/30/2012 4:47:13 PM3/30/2012 4:47:13 PM

Page 9: Clinical Scenarios in Surgery - Booksca.ca...Clinical Scenarios in Surgery DECISION MAKING AND OPERATIVE TECHNIQUE Editors Justin B. Dimick, MD, MPH Assistant Professor of Surgery

Contributors ix

Samuel R.G. Finlayson, MD, MPHKessler DirectorCenter for Surgery & Public HealthHarvard Medical SchoolOne Brigham CircleAssociate SurgeonDepartment of SurgeryBrigham & Women’s HospitalBoston, Massachusetts

Acute Appendicitis

Emily M. Fontenot, MDSurgical ResidentDepartment of SurgeryUniversity of North CarolinaUniversity of North Carolina HospitalChapel Hill, North Carolina

Omphalocele

Heidi L. Frankel, MDAssistant Professor of SurgeryDepartments of Surgery and Surgical Critical CareUniversity of Maryland Medical CetnerBaltimore, Maryland

Adrenal Insuffi ciency

Timothy L. Frankel, MDSurgical Oncology FellowDepartment of SurgeryMemorial Sloan-Kettering Cancer CenterNew York, New York

Extremity Mass (Sarcoma)Obstructive Jaundice

Michael G. Franz, MDVice President Global Clinical and Medical AffairsLifeCell CorporationBranchburg, New Jersey

Enterocutaneous Fistula

Danielle Fritze, MDHouse Offi cerDepartment of SurgeryUniversity of MichiganAnn Arbor, Michigan

Acute CholecystitisBleeding Gastric Ulcer

Guillermo A. Escobar, MDAssistant Professor of SurgeryVascular SurgeonUniversity of MichiganAnn Arbor, Michigan

Ruptured Abdominal Aortic Aneurysm

David A. Etzioni, MD, MSHSAssociate ProfessorDepartment of SurgeryMayo Clinic College of MedicineRochester, MinnesotaSenior Associate ConsultantDepartment of SurgeryMayo Clinic, ArizonaPhoenix, Arizona

Large Bowel Obstruction from Colon Cancer

Heather L. Evans, MD, MSAssistant ProfessorDepartment of SurgeryUniversity of WashingtonSeattle, Washington

Abdominal Compartment Syndrome

Gavin A. Falk, MDGeneral Surgery ResidentDepartment of General SurgeryCleveland Clinic FoundationCleveland, Ohio

Rectal Bleeding in a Young Child

Jonathan F. Finks, MDAssistant ProfessorDepartment of SurgeryUniversity of Michigan Health SystemAnn Arbor, Michigan

Gastroesophageal Refl ux DiseaseRecurrent Inguinal Hernia

Emily Finlayson, MD, MSAssistant ProfessorDepartment of SurgeryUniversity of California, San FranciscoSan Francisco, California

Crohn’s Disease with Small Bowel Stricture

Dimick_FM.indd ixDimick_FM.indd ix 3/30/2012 4:47:13 PM3/30/2012 4:47:13 PM

Page 10: Clinical Scenarios in Surgery - Booksca.ca...Clinical Scenarios in Surgery DECISION MAKING AND OPERATIVE TECHNIQUE Editors Justin B. Dimick, MD, MPH Assistant Professor of Surgery

x Contributors

Tyler Grenda, MDResident in General SurgeryDepartment of SurgeryUniversity of Michigan Health SystemAnn Arbor, Michigan

Achalasia

Erica R. Gross, MDResearch FellowPediatric SurgeryCollege of Physicians and Surgeons, Columbia

UniversityPediatric ECMO FellowPediatric SurgeryMorgan Stanley Children’s Hospital, New YorkNew York, New York

Emesis in an Infant

Travis E. Grotz, MDResidentGeneral SurgeryMayo ClinicRochester, Minneapolis

Palpable Breast Mass

Oliver L. Gunter, MDAssistant Professor of SurgeryBiomedical Research Education & TrainingVanderbilt University Medical CenterNashville, Tennessee

Pelvic Fracture

Jeffrey S. Guy, MD, MSc, MMHCAssociate ProfessorDepartment of SurgeryVanderbilt UniversityDirector-Regional Burn Center, Director-Acute

Operative ServicesDepartment of SurgeryVanderbilt University Medical CenterNashville, Tennessee

Burns

Adil H. Haider, MD, MPHAssociate Professor of SurgeryAnesthesiology and Health Policy and ManagementDirectorCenter for Surgical Trials and Outcomes Research

(CSTOR)Johns Hopkins HospitalDivision of Acute Care SurgeryBaltimore, Maryland

Penetrating Chest Injury

Samir K. Gadepalli, MDPediatric Surgery FellowDepartment of Pediatric SurgeryUniversity of MichiganCS Mott Children’s HospitalAnn Arbor, Michigan

Gastroschisis

Wolfgang B. Gaertner, MS, MDSurgical Chief ResidentDepartment of SurgeryUniversity of MinnesotaUniversity of Minnesota Medical CenterMinneapolis, Minnesota

Appendiceal Carcinoid Tumor

Paul G. Gauger, MDWilliam J. Fry Professor of SurgeryDepartment of SurgeryUniversity of MichiganAnn Arbor, Michigan

Adrenal Cancer

James D. Geiger, MDProfessor of SurgerySection of Pediatric SurgeryCS Mott Children’s HospitalUniversity of MichiganExecutive DirectorMedical Innovation CenterUniversity of MichiganAnn Arbor, Michigan

IntussusceptionGastroschisis

Philip P. Goodney, MD, MSAssistant ProfessorCenter for Health Policy ResearchThe Dartmouth InstituteHanover, New HampshireAssistant ProfessorDepartment of Surgery, Section of Vascular SurgeryDartmouth Hitchcock Medical CenterLebanon, New Hampshire

Lifestyle-Limiting Claudication

Sarah E. Greer, MD, MPHClinical FellowDepartment of SurgeryHospital of the University of PennsylvaniaPhiladelphia, Pennsylvania

Acute Appendicitis

Dimick_FM.indd xDimick_FM.indd x 3/30/2012 4:47:13 PM3/30/2012 4:47:13 PM

Page 11: Clinical Scenarios in Surgery - Booksca.ca...Clinical Scenarios in Surgery DECISION MAKING AND OPERATIVE TECHNIQUE Editors Justin B. Dimick, MD, MPH Assistant Professor of Surgery

Contributors xi

David W. Healy, MD, MRCP, FRCAAssistant ProfessorAnesthesiologyUniversity of MichiganDirector, Head & Neck AnesthesiaDepartment of Anesthesiology Health SystemsUniversity of Michigan Hospital and Health SystemsAnn Arbor, Michigan

Airway Emergency

Mark R. Hemmila, MDAssociate Professor of SurgeryAcute Care SurgeryUniversity of MichiganAnn Arbor, Michigan

Duodenal InjuryPerforated Appendicitis

Samantha Hendren, MD, MPHAssistant ProfessorDepartment of SurgeryUniversity of MichiganColorectal SurgeonDepartment of General SurgeryAnn Arbor VA Healthcare SystemAnn Arbor, Michigan

Medically Refractory Ulcerative Colitis

Peter K. Henke, MDProfessor of SurgerySurgeryUniversity of MichiganAnn Arbor, Michigan

Acute Limb Ischemia

Richard Herman, MDInstructor of SurgeryUniversity of Michigan Hospital and Health Systems Department of Pediatric SurgeryAnn Arbor, Michigan

Necrotizing Enterocolitis

Michael G. House, MDAssistant ProfessorDepartment of SurgeryIndiana University School of MedicineIndianapolis, Indiana

Symptomatic Pancreatic Pseudocyst

Ihab HalaweishHouse Offi cer IIDepartment of SurgeryUniversity of MichiganAnn Arbor, Michigan

Neuroblastoma

A.L. HalversonAssociate Professor of SurgeryNorthwestern UniversityFeinberg School of MedicineDepartment of SurgeryChicago, Illinois

Anal Carcinoma

Allen Hamdan, MDAssociate Professor of SurgeryDepartment of SurgeryHarvard Medical SchoolAttending SurgeonDivision of Vascular and Endovascular SurgeryBeth Israel Deaconess Medical CenterBoston, Massachusetts

Diabetic Foot Infection

James Harris Jr., MDSurgical ResidentDepartment of SurgeryJohns Hopkins HospitalBaltimore, Maryland

Solitary Pulmonary Nodule

Elliott R. Haut, MDAssociate Professor of SurgeryAnesthesiology/Critical Care Medicine (ACCM) and

Emergency MedicineDivision of Acute Care Surgery, Department of

SurgeryThe Johns Hopkins University School of MedicineDirectorTrauma/Acute Care Surgery FellowshipThe Johns Hopkins HospitalBaltimore, Maryland

Nutritional Support in the Critically Ill Surgery Patient

A.V. HaymanGeneral Surgical ResidentNorthwestern Memorial HospitalChicago, Illinois

Anal Carcinoma

Dimick_FM.indd xiDimick_FM.indd xi 3/30/2012 4:47:14 PM3/30/2012 4:47:14 PM

Page 12: Clinical Scenarios in Surgery - Booksca.ca...Clinical Scenarios in Surgery DECISION MAKING AND OPERATIVE TECHNIQUE Editors Justin B. Dimick, MD, MPH Assistant Professor of Surgery

xii Contributors

Angela M. Ingraham, MD, MSGeneral Surgery ResidentDepartment of SurgeryUniversity of CincinnatiCincinnati, Ohio

Postoperative Dehiscence

Kamal M.F. Itani, MDProfessor of SurgeryDepartment of SurgeryBoston UniversityBoston, MassachusettsChief of SurgeryDepartment of SurgeryVA Boston Health Care SystemWorcester, Massachusetts

Ventral Incisional Hernias

Alexis D. Jacob, MDVascular Surgery FellowDivision of Vascular Surgery and Endovascular

SurgeryUniversity of Florida College of MedicineGainesville, Florida

Need for Hemodialysis Access

Lisa K. Jacobs, MDAssistant Professor of SurgeryDirector of Clinical Breast Cancer ResearchDepartments of Surgery and OncologyJohns Hopkins UniversityBaltimore, Maryland

Suspicious Mammographic Abnormality

James W. Jakub, MDAssistant Professor of SurgeryGeneral SurgeryMayo ClinicRochester, Minnesota

Palpable Breast Mass

Jennifer E. Joh, MDThe Hoffberger Breast Center at MercyBaltimore, Maryland

Breast Cancer During Pregnancy

Gina M.S. Howell, MDSurgery ResidentDepartment of SurgeryUniversity of PittsburghPittsburgh HospitalUniversity of Pittsburgh Medical CenterPittsburgh, Pennsylvania

Stab Wound to the Neck

Thomas S. Huber, MD, PhDProfessor and ChiefDivision of Vascular Surgery and Endovascular

SurgeryUniversity of Florida College of MedicineGainesville, Florida

Need for Hemodialysis Access

David T. Hughes, MDAssistant ProfessorDepartment of SurgeryAlbert Einstein College of MedicineAttending SurgeonDepartment of SurgeryMontefi ore Medical CenterBronx, New York

Adrenal Cancer

Alicia HulbertClinical FellowDepartment of OncologySchool of Medicine Baltimore, Maryland

Solitary Pulmonary Nodule

Justin Hurie, MDAssistant ProfessorDepartment of Vascular and Endovascular SurgeryWake Forest UniversityAttending SurgeonDepartment of Vascular and Endovascular SurgeryNorth Carolina Baptist HospitalWinston-Salem, North Carolina

Deep Venous Thrombosis

Neil Hyman, MDSamuel B. and Michelle D. Labow Professor of

SurgeryCodirector, Digestive Disease CenterDepartment of SurgeryUniversity of Vermont College of MedicineBurlington, Vermont

Anastomotic Leak After Colectomy

Dimick_FM.indd xiiDimick_FM.indd xii 3/30/2012 4:47:14 PM3/30/2012 4:47:14 PM

Page 13: Clinical Scenarios in Surgery - Booksca.ca...Clinical Scenarios in Surgery DECISION MAKING AND OPERATIVE TECHNIQUE Editors Justin B. Dimick, MD, MPH Assistant Professor of Surgery

Contributors xiii

Sajid A. Khan, MDSurgical Oncology FellowDepartment of SurgeryThe University of Chicago Medical CenterChicago, Illinois

Refractory Pain from Chronic Pancreatitis

Hyaehwan Kim, MDSurgery ResidentBrookdale University Hospital and Medical CenterNewyork

Rectal Cancer

Andrew S. Klein, MD, MBAProfessorDepartment of SurgeryDirectorComprehensive Transplant CenterCedars Sinai Medical CenterLos Angeles, California

Variceal Bleeding and Portal Hypertension

Carla Kohoyda-Inglis, MPAProgram DirectorInternational Center for Automotive MedicineAnn Arbor, Michigan

Blunt Abdominal Trauma from Motor Vehicle Crash

Geoffrey W. Krampitz, MDGeneral Surgery ResidentDepartment of SurgeryStanford Hospital and ClinicsStanford, California

Gastrinoma

Andrew Kroeker, MDResident SurgeonDepartment of OtolaryngologyUniversity of MichiganAnn Arbor, Michigan

Melanoma of the Head and Neck

Hari R. Kumar, MDChief ResidentDepartment of SurgeryIndiana University School of MedicineIndianapolis, Indiana

Cortisol-secreting Adrenal Tumor

Jussuf T. Kaifi , MD, PhDAssistant Professor of Surgery and MedicineDepartment of SurgeryPenn State College of MedicineAssistant Professor of Surgery and MedicineDepartment of SurgeryPenn State Hershey Medical CenterHershey, Pennsylvania

Gastric Cancer

Jeffrey Kalish, MDLaszlo N. Tauber Assistant ProfessorDepartment of SurgeryBoston University School of MedicineDirector of Endovascular SurgeryDepartment of SurgeryBoston Medical CenterBoston, Massachusetts

Diabetic Foot Infection

Lillian S. Kao, MD, MSAssociate ProfessorDepartment of SurgeryUniversity of Texas Health Science Center at HoustonHouston, TexasVice-ChiefDepartment of SurgeryLBJ General HospitalHouston, Texas

Necrotizing Soft Tissue Infections

Muneera R. Kapadia, MDClinical Assistant ProfessorDepartment of SurgeryUniversity of Iowa Hospitals and ClinicsIowa City, Iowa

Ischemic Colitis

Srinivas Kavuturu, MD, FRCSAssistant Professor of SurgeryDepartment of SurgeryMichigan State University, College of Human

MedicineAttending PhysicianDepartment of SurgerySparrow HospitalLansing, Michigan

Gastric Cancer

Dimick_FM.indd xiiiDimick_FM.indd xiii 3/30/2012 4:47:14 PM3/30/2012 4:47:14 PM

Page 14: Clinical Scenarios in Surgery - Booksca.ca...Clinical Scenarios in Surgery DECISION MAKING AND OPERATIVE TECHNIQUE Editors Justin B. Dimick, MD, MPH Assistant Professor of Surgery

xiv Contributors

Pamela A. Lipsett, MD, MHPEWarfi eld M Firor Professor of SurgeryProgram DirectorGeneral Surgery and Surgical Critical CareCo-Director of the Surgical Intensive Care UnitsJohns Hopkins HospitalBaltimore, Maryland

Septic Shock

Ann C. Lowry, MDClinical Professor of SurgeryDivision of Colon and Rectal SurgeryUniversity of MinnesotaSt. Paul, Minnesota

Ischemic Colitis

Dennis P. Lund, MDSurgeon-in-Chief, Phoenix Children’s HospitalExecutive Vice President, Phoenix Children’s Medical

GroupProfessor, Department of Child HealthAcademic Division Chief, Pediatric Surgery,

Department of Child HealthUniversity of Arizona College of Medicine—PhoenixPhoenix, Arizona

Malrotation and Midgut Volvulus

Paul M. Maggio, MD, MBAAssistant Professor of SurgeryDepartment of SurgeryStanford UniversityAssociate Director of TraumaDepartment of SurgeryStanford University HospitalStanford, California

Symptomatic Cholelithiasis in Pregnancy

Ali F. Mallat, MD, MSAssistant Professor of SurgeryDepartment of General SurgeryUniversity Of MichiganGeneral Surgery Service ChiefDepartment of SurgeryAnn Arbor VA Medical CenterAnn Arbor, Michigan

Cholecystoduodenal fi stula

Sean T. Martin, MDAssociate Staff SurgeonColorectal SurgeryCleveland ClinicCleveland, Ohio

Complicated Diverticulitis

Adriana Laser, MDResident in Surgery University of Maryland Baltimore, Maryland

Ruptured Abdominal Aortic Aneurysm

Christine L. Lau, MDAssociate ProfessorSurgery, Thoracic & CardiovascularUniversity of Virginia Health SystemCharlottesville, Virginia

Esophageal Perforation

Constance W. Lee, MDSurgical ResidentDepartment of SurgeryUniversity of Florida College of MedicineGainesville, Florida

Perforated Duodenal Ulcer

Marie Catherine Lee, MDAssistant Professor of SciencesDivision of Oncologic ScienceUniversity of South FloridaAssistant MemberWomen’s Oncology—Breast DivisionMoffi tt Cancer CenterTampa, Florida

Breast Cancer During Pregnancy

Jules Lin, MDAssistant ProfessorSection of Thoracic SurgeryUniversity of Michigan Medical SchoolAssistant ProfessorSection of Thoracic SurgeryUniversity of Michigan Health SystemAnn Arbor, Michigan

Achalasia

Peter H. Lin, MDProfessor of SurgeryChief of Division of Vascular Surgery and

Endovascular TherapyMichael E. DeBakey Department of SurgeryBaylor College of MedicineHouston, Texas

Symptomatic Carotid Stenosis

Dimick_FM.indd xivDimick_FM.indd xiv 3/30/2012 4:47:14 PM3/30/2012 4:47:14 PM

Page 15: Clinical Scenarios in Surgery - Booksca.ca...Clinical Scenarios in Surgery DECISION MAKING AND OPERATIVE TECHNIQUE Editors Justin B. Dimick, MD, MPH Assistant Professor of Surgery

Contributors xv

Genevieve Melton-Meaux, MD, MAAssistant ProfessorDepartment of Surgery, Institute for Health

InformaticsUniversity of MinnesotaStaff SurgeonUniversity of Minnesota Medical CenterMinneapolis, Minnesota

Appendiceal Carcinoid Tumor

April E. Mendoza, MDTrauma Research FellowDepartment of SurgeryUniversity of North CarolinaDepartment of SurgeryUNC Memorial HospitalChapel Hill, North Carolina

Acute Renal Failure

Evangelos Messaris, MD, PhDAssistant ProfessorDivision of Colon and Rectal SurgeryPennsylvania State UniversityFacultyDivision of Colon and Rectal SurgeryMilton S. Hershey Medical CenterHershey, Pennsylvania

Symptomatic Primary Inguinal Hernia

Stacey A. Milan, MDAssistant Professor of SurgeryGeneral SurgeryJefferson Medical CollegeAssistant Professor of SurgeryGeneral SurgeryThomas Jefferson University HospitalPhiladelphia, Pennsylvania

Pancreatic Neuroendocrine Tumors

Barbra S. Miller, MDAssistant ProfessorSurgeryUniversity of MichiganAnn Arbor, Michigan

Primary Hyperaldosteronism

Judiann Miskulin, MDAssistant Professor of SurgeryEndocrine SurgeryDepartment of SurgeryIndiana University HealthIndianapolis, Indiana

Cortisol-secreting Adrenal Tumor

Jeffrey B. Matthews, MDDallas B. Phemister Professor of SurgeryChairman, Department of SurgerySurgery-In-ChiefDepartment of SurgeryThe University of ChicagoChicago, Illinois

Refractory Pain from Chronic Pancreatitis

Haggi Mazeh, MDClinical InstructorSurgeryUniversity of WisconsinMadison, Wisconsin

Palpable Thyroid Nodule

Timothy W. McCardle, MDAssistant MemberPathologyMoffi tt Cancer CenterTampa, Florida

Melanoma

Erin McKean, MDAssistant ProfessorDepartment of Otolaryngology—Head and Neck

SurgeryUniversity of Michigan Medical SchoolAssistant ProfessorDepartment of Otolaryngology—Head and Neck

SurgeryUniversity of Michigan Health SystemAnn Arbor, Michigan

Melanoma of the Head and NeckHead and Neck Cancer

Sean E. McLean, MDAssistant Professor of SurgeryDepartment of SurgeryUniversity of North Carolina at Chapel HillStaff SurgeonDepartment of SurgeryUNC HospitalsChapel Hill, North Carolina

Omphalocele

Michelle K. McNutt, MDAssistant Professor of SurgerySurgeryUniversity of Texas Medical School at HoustonHouston, Texas

Necrotizing Soft Tissue Infections

Dimick_FM.indd xvDimick_FM.indd xv 3/30/2012 4:47:14 PM3/30/2012 4:47:14 PM

Page 16: Clinical Scenarios in Surgery - Booksca.ca...Clinical Scenarios in Surgery DECISION MAKING AND OPERATIVE TECHNIQUE Editors Justin B. Dimick, MD, MPH Assistant Professor of Surgery

xvi Contributors

Lena M. Napolitano, MDProfessorDepartment of SurgeryUniversity of MichiganDivision Chief, Acute Care SurgeryDirector, Trauma & Surgical Critical CareDepartment of SurgeryUniversity of MichiganAnn Arbor, Michigan

Acute Respiratory Distress Syndrome (ARDS)

Avery B. Nathens, MD, MPH, PhDProfessorDepartment of SurgeryUniversity of TorontoDivision Head in General SurgeryDirector of TraumaGeneral Surgery & TraumaSt. Michael’s HospitalToronto, Ontario

Postoperative Dehiscence

Erika Newman, MDAssistant Professor in Pediatric SurgeryEdith Briskin Emerging ScholarCS Mott Children’s HospitalDepartment of SurgeryA. Alfred Taubman Medical Research InstituteThe University of Michigan Medical SchoolAnn Arbor, Michigan

Neuroblastoma

Lisa A. Newman, MD, MPHProfessor of SurgeryDirector, Breast Care CenterDepartment of SurgeryAnn Arbor, Michigan

Lobular Carcinoma In Situ

Jeffrey A. Norton, MDProfessor of SurgeryChief, Division of General SurgeryStanford University Medical CenterStanford, California

Gastrinoma

Babak J. Orandi, MD, MScGeneral Surgery ResidentDepartment of SurgeryJohns Hopkins UniversityJohns Hopkins HospitalBaltimore, Maryland

Acute Mesenteric Ischemia

Derek Moore, MD, MPHAssistant Professor of SurgeryDepartment of Surgery, Division of Liver, Kidney and

Pancreas TransplantationVanderbilt University Medical CenterNashville, Tennessee

End-Stage Renal Disease (Renal Transplantation)

Arden M. Morris, MDAssociate ProfessorDepartment of SurgeryChief, Division of ColorectalSurgeryUniversity of MichiganAnn Arbor, Michigan

Colonic Vovulus

Monica Morrow, MDProfessor of SurgeryWeill Medical College of Cornell UniversityNew York, New YorkChief, Breast ServiceDepartment of SurgeryAnne Burnett Windfohr Chair of Clinical OncologyMemorial Sloan-Kettering Cancer CenterNew York, New York

Infl ammatory Breast Cancer

John Morton, MD, MPHAssociate Professor of SurgeryDirector of Bariatric SurgeryDepartment of SurgeryStanford UniversityStanford, California

Morbid Obesity

Michael Mulholland, MD, PhDProfessor and ChairDepartment of SurgeryUniversity of MichiganSurgeon in ChiefUniversity of Michigan HospitalAnn Arbor, Michigan

Bleeding Gastric Ulcer

Alykhan S. Nagji, MDResidentDepartment of SurgeryUniversity of Virginia Hospital SystemCharlottesville, Virginia

Esophageal Perforation

Dimick_FM.indd xviDimick_FM.indd xvi 3/30/2012 4:47:14 PM3/30/2012 4:47:14 PM

Page 17: Clinical Scenarios in Surgery - Booksca.ca...Clinical Scenarios in Surgery DECISION MAKING AND OPERATIVE TECHNIQUE Editors Justin B. Dimick, MD, MPH Assistant Professor of Surgery

Contributors xvii

Catherine E. Pesce, MDDepartment of Medical OncologyThe Sidney Kimmel Comprehensive Cancer Center

at Johns HopkinsBaltimore, Maryland

Suspicious Mammographic Abnormality

Rebecca Plevin, MDDepartment of SurgeryUniversity of Washington Medical CenterSeattle, Washington

Abdominal Compartment Syndrome

Benjamin K. Poulose, MD, MPHAssistant ProfessorDepartment of SurgeryVanderbilt University School of MedicineAssociate Director, Endoscopy SuiteDepartment of SurgeryVanderbilt University Medical CenterNashville, Tennessee

Cholangitis

Sandhya Pruthi, MDAssociate Professor of MedicineGeneral Internal MedicineMayo ClinicRochester, Minnesota

Palpable Breast Mass

Krishnan Raghavendran, MDAssociate ProfessorSurgeryUniversity of Michigan Hospital and Health

SystemsAnn Arbor, Michigan

Ventilator-Associated PneumoniaAcute Respiratory Distress Syndrome (ARDS)

Matthew W. Ralls, MDSurgical House Offi cerDepartment of SurgeryUniversity of MichiganAnn Arbor, Michigan

Incarcerated/Strangulated Inguinal Hernia

Mark B. Orringer, MDProfessor of SurgerySection of Thoracic SurgeryUniversity of MichiganAnn Arbor, Michigan

Esophageal Cancer

Paul Park, MD, MAChief ResidentDepartment of SurgerySection of General SurgeryUniversity of Michigan Medical SchoolUniversity of Michigan HospitalAnn Arbor, Michigan

Cholecystoduodenal fi stula

Pauline K. Park, MDAssociate ProfessorDepartment of SurgeryUniversity of MichiganCo-Director Surgical Intensive Care UnitDepartment of SurgeryUniversity of Michigan Health SystemAnn Arbor, Michigan

Acute Respiratory Distress Syndrome (ARDS)

Timothy M. Pawlik, MD, MPHAssociate ProfessorDepartment of SurgeryJohns Hopkins UniversityJohns Hopkins HospitalBaltimore, Maryland

Metastatic Colorectal Cancer

Shawn J. Pelletier, MDAssociate ProfessorSurgical Director of Liver TransplantationDepartment of SurgeryUniversity of Michigan Health SystemAnn Arbor, Michigan

Incidental Liver Mass

Peter D. Peng, MD, MSSurgical Oncology FellowDepartment of SurgeryJohns Hopkins HospitalBaltimore, Maryland

Metastatic Colorectal Cancer

Dimick_FM.indd xviiDimick_FM.indd xvii 3/30/2012 4:47:14 PM3/30/2012 4:47:14 PM

Page 18: Clinical Scenarios in Surgery - Booksca.ca...Clinical Scenarios in Surgery DECISION MAKING AND OPERATIVE TECHNIQUE Editors Justin B. Dimick, MD, MPH Assistant Professor of Surgery

xviii Contributors

Michael J. Rosen, MDAssociate Professor of SurgeryChief Division of GI and General SurgeryDepartment of SurgeryCase Western Reserve UniversityCleveland, Ohio

Complex Abdominal Wall Reconstruction

Michael S. Sabel, MDAssociate ProfessorSurgeryUniversity of MichiganAnn Arbor, Michigan

Melanoma Presenting with Regional Lymph Node InvolvementMerkel Cell Carcinoma

Vivian M. Sanchez, MDAssistant Professor of SurgeryDepartment of SurgeryBoston UniversityBoston, MassachusettsMinimally Invasive and Bariatric SurgeryDepartment of SurgeryVA Boston Health Care SystemWest Roxbury, Massachusetts

Ventral Incisional Hernias

George A. Sarosi Jr., MDAssociate Professor of SurgeryDepartment of SurgeryUniversity of Florida College of MedicineStaff SurgeonSurgical ServiceNorth Florida/South Georgia VA Medical CenterGainesville, Florida

Perforated Duodenal Ulcer

Brian D. Saunders, MDAssistant ProfessorDepartments of Surgery and MedicinePenn State College of MedicineAssistant ProfessorDepartments of Surgery and MedicinePenn State Milton S. Hershey Medical CenterHershey, Pennsylvania

Incidental Adrenal Mass

John E. Rectenwald, MDAssociate Professor of Surgery & RadiologyProgram Director, Vascular SurgerySection of Vascular SurgeryUniversity of Michigan Health SystemAnn Arbor, Michigan

Asymptomatic Carotid Stenosis

John W. Rectenwald, MDAssociate Professor of SurgerySurgeryUniversity of MichiganAnn Arbor, Michigan

Acute Limb Ischemia

Scott E. Regenbogen, MD, MPHAssistant ProfessorDepartment of SurgeryUniversity of MichiganAnn Arbor, MichiganStaff SurgeonDepartment of SurgeryUniversity of Michigan Health SystemAnn Arbor, Michigan

Lower Gastrointestinal Bleeding

Amy L. Rezak, MDAssistant ProfessorDepartment of SurgeryUniversity of North Carolina at Chapel Hill School of

MedicineTrauma, Critical Care SurgeonDepartment of SurgeryUNC Health CareChapel Hill, North Carolina

Severe Acute Pancreatitis

William P. Robinson III, MDAssistant Professor of SurgeryDivision of Vascular and Endovascular SurgeryUniversity of Massachusetts Medical SchoolDivision of Vascular and Endovascular SurgeryUMass Memorial Medical CenterWorcester, Massachusetts

Tissue Loss Due to Arterial Insuffi ciency

Dimick_FM.indd xviiiDimick_FM.indd xviii 3/30/2012 4:47:14 PM3/30/2012 4:47:14 PM

Page 19: Clinical Scenarios in Surgery - Booksca.ca...Clinical Scenarios in Surgery DECISION MAKING AND OPERATIVE TECHNIQUE Editors Justin B. Dimick, MD, MPH Assistant Professor of Surgery

Contributors xix

Rebecca S. Sippel, MDAssistant ProfessorDepartment of SurgeryUniversity of WisconsinChief of Endocrine SurgeryDepartment of SurgeryUniversity of Wisconsin Hospitals and ClinicsMadison, Wisconsin

Palpable Thyroid Nodule

Alexis D. Smith, MDGeneral Surgery ResidentDepartment of General SurgeryUniversity of Maryland School of MedicineBaltimore, Maryland

Pheochromocytoma

Vance L. Smith, MD, MBAAttending Surgeon/Surgical IntensivistDepartment of SurgeryDivision of Trauma SurgeryEden Medical Center—SutterhealthCastro Valley, California

Symptomatic Cholelithiasis in Pregnancy

Oliver S. Soldes, MDStaff Pediatric SurgeonDepartment of Pediatric SurgeryCleveland Clinic FoundationCleveland, Ohio

Rectal Bleeding in a Young Child

Vernon K. Sondak, MDProfessorSurgery and Oncologic SciencesUniversity of South FloridaDepartment ChairCutaneous OncologyH. Lee Moffi tt Cancer Center & Research InstituteTampa, Florida

Melanoma

Christopher J. Sonnenday, MD, MHSAssistant Professor of SurgeryAssistant Professor of Health Management & PolicyUniversity of MichiganAnn Arbor, Michigan

Bile Duct InjuryLiver Mass in Chronic Liver DiseaseObstructive Jaundice

C. Max Schmidt, MD, PhD, MBAAssociate ProfessorSurgeryIndiana University School of MedicineIndianapolis, Indiana

Incidental Pancreatic Cyst

Maureen K. Sheehan, MDAssistant ProfessorDivision of Vascular SurgeryUniversity of Texas Health Science Center at

San AntonioSan Antonio, Texas

Chronic Mesenteric Ischemia

Terry Shih, MDHouse Offi cerDepartment of SurgeryUniversity of MichiganUniversity of Michigan Health SystemAnn Arbor, Michigan

Perforated Appendicitis

Andrew Shuman, MDChief ResidentDepartment of Otolaryngology—Head and Neck

SurgeryUniversity of Michigan HospitalsAnn Arbor, Michigan

Melanoma of the Head and Neck

Sabina Siddiqui, MDPediatric Surgical Critical Care FellowDepartment of Pediatric SurgeryUniversity of Michigan, Ann ArborFellowDepartment of Pediatric SurgeryC.S. Mott’s Children’s HospitalAnn Arbor, Michigan

Intussusception

Matthew J. Sideman, MDAssociate ProfessorDepartment of SurgeryUniversity of Texas Health Science Center

at San AntonioSan Antonio, Texas

Chronic Mesenteric Ischemia

Dimick_FM.indd xixDimick_FM.indd xix 3/30/2012 4:47:14 PM3/30/2012 4:47:14 PM

Page 20: Clinical Scenarios in Surgery - Booksca.ca...Clinical Scenarios in Surgery DECISION MAKING AND OPERATIVE TECHNIQUE Editors Justin B. Dimick, MD, MPH Assistant Professor of Surgery

xx Contributors

Pierre TheodoreAssociate ProfessorVan Auken Chair in Thoracic SurgeryUCSF Medical CenterSan Francisco, California

Spontaneous Pneumothorax

Thadeus TrusAssociate Professor of SurgeryDepartment of SurgeryDartmouth Medical School Lebanon, New Hampshire

Paraesophageal Hernia

Douglas J. Turner, MDAssociate ProfessorDepartment of SurgeryUniversity of Maryland School of MedicineChief, General SurgeryBaltimore VA Medical CenterBaltimore, Maryland

Pheochromocytoma

Gilbert R. Upchurch Jr., MDWilliam H. Muller, Jr. ProfessorChief of Vascular and Endovascular SurgeryUniversity of VirginiaCharlottesville, Virginia

Pulsatile Abdominal MassRuptured Abdominal Aortic AneurysmAsymptomatic Carotid Stenosis

Kyle J. Van Arendonk, MDHalsted ResidentDepartment of SurgeryJohns Hopkins HospitalBaltimore, Maryland

Nutritional Support in the Critically Ill Surgery Patient

Chandu Vemuri, MDVascular Surgery FellowDepartment of SurgeryWashington University in St. LouisSt. Louis, Missouri

Retroperitoneal Sarcoma

Jon D. Vogel, MDStaff Colorectal SurgeonCleveland ClinicCleaveland, Ohio

Complicated Diverticulitis

Julie Ann Sosa, MD, MAAssociate Professor of Surgery and Medicine

(Oncology)Dept of Surgery, Divisions of Endocrine Surgery and

Surgical OncologyYale University School of MedicineNew Haven, Connecticut

Primary Hyperparathyroidism

Matthew Spector, MDChief ResidentDepartment of OtolaryngologyUniversity of MichiganAnn Arbor, Michigan

Head and Neck Cancer

Jason L. Sperry, MD, MPHAssistant Professor of Surgery and Critical Care

MedicineUniversity of Pittsburgh Medical CenterPittsburgh, Pennsylvania

Stab Wound to the Neck

Kevin F. Staveley-O’Carroll, MD, PhDProfessor of Surgery, Medicine, Microbiology

and ImmunologyDepartment of SurgeryPenn State College of MedicinePenn State Hershey Medical CenterHershey, Pennsylvania

Gastric Cancer

John F. Sweeney, MDW. Dean Warren Distinguished Professor of SurgeryDepartment of SurgeryEmory University School of MedicineAtlanta, Georgia

Splenectomy for Hematologic Disease

Kevin E. Taubman, MDAssistant Professor Department of SurgeryUniversity of Oklahoma College of MedicineTulsa, Oklahoma

Chronic Mesenteric Ischemia

Daniel H. Teitelbaum, MDProfessorDepartment of Surgery, Section of Pediatric SurgeryUniversity of MichiganAnn Arbor, Michigan

Necrotizing Enterocolitis

Dimick_FM.indd xxDimick_FM.indd xx 3/30/2012 4:47:14 PM3/30/2012 4:47:14 PM

Page 21: Clinical Scenarios in Surgery - Booksca.ca...Clinical Scenarios in Surgery DECISION MAKING AND OPERATIVE TECHNIQUE Editors Justin B. Dimick, MD, MPH Assistant Professor of Surgery

Contributors xxi

Walter P. Weber, MDAssistant ProfessorDepartment of SurgeryUniversity of BaselAttending SurgeonDepartment of SurgeryUniversity Hospital of BaselBasel, Switzerland

Infl ammatory Breast Cancer

Martin R. Weiser, MDAssociate MemberSurgeryMemorial Sloan-Kettering Cancer CenterNew York, New YorkAssociate ProfessorSurgeryCornell Weill Medical School/New York Presbyterian

HospitalNew York, New York

Rectal Cancer

Bradford P. Whitcomb, MDAssociate Residency Program DirectorDepartment of Obstetrics and GynecologyTripler Army Medical CenterHonolulu, Hawaii

Gynecologic Causes of Lower Abdominal Pain

Elizabeth C. Wick, MDAssistant ProfessorDepartment of SurgeryJohns Hopkins UniversityStaff SurgeonDepartment of SurgeryJohns Hopkins Medical InstitutionsBaltimore, Maryland

Appendiceal Carcinoid Tumor

Sandra L. Wong, MD, MSAssistant ProfessorDepartment of SurgeryUniversity of MichiganAnn Arbor, MichiganStaff PhysicianDepartment of SurgeryUniversity of Michigan Hospital and Health SystemsAnn Arbor, Michigan

Retroperitoneal Sarcoma

Wendy L. Wahl, MDProfessor of SurgeryDepartment of SurgeryUniversity of Michigan Health SystemAnn Arbor, Michigan

Bleeding Duodenal Ulcer

Thomas W. Wakefi eld, MDS. Martin Lindenauer Professor of SurgeryDepartment of Vascular SurgeryUniversity of MichiganHead, Section of Vascular SurgeryDepartment of Vascular SurgeryUniversity of Michigan Health SystemsAnn Arbor, Michigan

Deep Venous Thrombosis

Jennifer F. Waljee, MD, MSHouse Offi cerDepartment of SurgeryUniversity of MichiganUniversity of Michigan Medical CenterAnn Arbor, Michigan

Breast Reconstruction

Stewart C. Wang, MD, PhDEndowed Professor of SurgeryDirector, International Center for Automotive

MedicineUniversity of MichiganAnn Arbor, Michigan

Blunt Abdominal Trauma from Motor Vehicle Crash

Joshua A. Waters, MDResidentDepartment of SurgeryIndiana University School of MedicineIndianapolis, Indiana

Incidental Pancreatic Cyst

Sarah M. Weakley, MDMichael E. DeBakey Department of SurgeryBaylor College of MedicineHouston, Texas

Symptomatic Carotid Stenosis

Dimick_FM.indd xxiDimick_FM.indd xxi 3/30/2012 4:47:14 PM3/30/2012 4:47:14 PM

Page 22: Clinical Scenarios in Surgery - Booksca.ca...Clinical Scenarios in Surgery DECISION MAKING AND OPERATIVE TECHNIQUE Editors Justin B. Dimick, MD, MPH Assistant Professor of Surgery

xxii Contributors

Charles J. Yeo, MDSamuel D. Gross Professor and ChairmanDepartment of SurgeryThomas Jefferson UniversityChiefDepartment of SurgeryThomas Jefferson University HospitalPhiladelphia, Pennsylvania

Pancreatic Neuroendocrine Tumors

Barbara Zarebczan, MDGeneral Surgery ResidentSurgeryUniversity of WisconsinMadison, Wisconsin

Medullary Thyroid Cancer

Derek T. Woodrum, MDAssistant ProfessorDepartment of AnesthesiologyUniversity of Michigan Medical SchoolFaculty AnesthesiologistDepartment of AnesthesiologyUniversity of Michigan Medical CenterAnn Arbor, Michigan

Airway Emergency

Leslie S. Wu, MDAttending surgeonDepartment of SurgeryMaine Medical CenterPortland, Maine

Primary Hyperparathyroidism

Dimick_FM.indd xxiiDimick_FM.indd xxii 3/30/2012 4:47:14 PM3/30/2012 4:47:14 PM

Page 23: Clinical Scenarios in Surgery - Booksca.ca...Clinical Scenarios in Surgery DECISION MAKING AND OPERATIVE TECHNIQUE Editors Justin B. Dimick, MD, MPH Assistant Professor of Surgery

xxiii

Foreword

In preparing a generation of surgical residents to enter practice, there are some point-ers that I may offer. There are also some rules that I have picked up while writing and editing chapters for surgical textbooks. Most of us are not born surgeons. If you are the

exception—accomplished, articulate, and confi dent; if surgical principles come effortlessly, you may stop reading now. Still, you might want to take a look. Here are three thoughts:

1. Start reading right away.

For most surgeons, the most diffi cult reading assignment is the fi rst assignment. The problem lies not in realizing the high stakes of a board exam; the trouble comes with the commitment that board preparation requires. The form of most contemporary texts is part of the problem. A glance shows the chapters to be long, devoid of illustrations, a tex-tual sensory deprivation. Clinical Scenarios in Surgery is so inviting with its crisp writing, generous illustrations, and telegenic presentation that it begs to be read. Get started.

2. Grab hold of the present and look to the future.

Modern surgery is forward looking, seeking to improve the care of current patients and to prevent disease in potential future patients. Given the pace of modern biomedi-cal research, no lone individual can be expected to fi nd, read, synthesize, and apply all new knowledge relevant to any clinical problem. All surgeons need an occasional guide through the surgical literature. In the midst of this information overload, the experienced, energetic editors of Clinical Scenarios in Surgery strike just the right balance. Keep going.

3. Keep reading, even just a little bit, every day.

Reading is a skill, sharpened with practice, perfected by continuous practice. Operative surgery reinforces this notion. The physical skills, sense of prioritized organization, personal confi dence, and intuition of the accomplished surgeon result from attention to the craft. That is the reason it is called the practice of surgery. Like the scalpel, a book becomes much friendlier with frequent use. Enjoy the journey.

Michael W. Mulholland, M.D., Ph.D.

Dimick_FM.indd xxiiiDimick_FM.indd xxiii 3/30/2012 4:47:14 PM3/30/2012 4:47:14 PM

Page 24: Clinical Scenarios in Surgery - Booksca.ca...Clinical Scenarios in Surgery DECISION MAKING AND OPERATIVE TECHNIQUE Editors Justin B. Dimick, MD, MPH Assistant Professor of Surgery

Dimick_FM.indd xxivDimick_FM.indd xxiv 3/30/2012 4:47:14 PM3/30/2012 4:47:14 PM

This page intentionally left blank.

Page 25: Clinical Scenarios in Surgery - Booksca.ca...Clinical Scenarios in Surgery DECISION MAKING AND OPERATIVE TECHNIQUE Editors Justin B. Dimick, MD, MPH Assistant Professor of Surgery

Preface

Despite remarkable technical advances and rapid scientifi c progress, it has never been more challenging to become a safe and profi cient surgeon.

Young surgeons are challenged by both the pace of new information and the sub-specialization occurring in every surgical discipline. Traditional surgical textbooks, which have grown to keep pace with these changes, are becoming encyclopedic reference books, which we turn to only when we need a comprehensive overview. With the vast amount of information available, it is often diffi cult to sort out the basic principles of safe surgery for a given clinical scenario. The mismatch between existing education materials and the need for a solid understanding of general surgical principles becomes most apparent when young surgeons sit down to prepare to take their written and oral board exams.

Young surgeons also learn differently than those in the past. Modern surgical trainees do not sit down and read for hours at a time. They are multitaskers who demand effi -ciency and immediate relevance in their learning materials. Most medical schools have responded to these changes by transitioning to curricula based on case-based learning. Clinical narratives are extremely effective learning tools because they use patient stories to teach essential surgical principles. Most existing surgical textbooks have not kept pace with these broader changes in medical education.

We wrote this book to fi ll these gaps. We have created a case-based text that communi-cates core principles of general surgery and its specialties. We believe the patient stories in these clinical scenarios will provide context to facilitate learning the principles of safe surgical care. Students, residents, and other young surgeons should fi nd the chapters short enough to read between cases or after a long day in the hospital. We hope this book will be particularly useful for senior surgical residents and recent graduates as they prepare for the American Board of Surgery oral examination.

Justin B. DimickGilbert R. Upchurch, Jr.

Christopher J. Sonnenday

xxv

Dimick_FM.indd xxvDimick_FM.indd xxv 3/30/2012 4:47:14 PM3/30/2012 4:47:14 PM

Page 26: Clinical Scenarios in Surgery - Booksca.ca...Clinical Scenarios in Surgery DECISION MAKING AND OPERATIVE TECHNIQUE Editors Justin B. Dimick, MD, MPH Assistant Professor of Surgery

Dimick_FM.indd xxviDimick_FM.indd xxvi 3/30/2012 4:47:15 PM3/30/2012 4:47:15 PM

This page intentionally left blank.

Page 27: Clinical Scenarios in Surgery - Booksca.ca...Clinical Scenarios in Surgery DECISION MAKING AND OPERATIVE TECHNIQUE Editors Justin B. Dimick, MD, MPH Assistant Professor of Surgery

Contents

Contributors vForeword xxiiiPreface xxv

AbdominalChapter 1Symptomatic Primary Inguinal Hernia 1Evangelos Messaris

Chapter 2Recurrent Inguinal Hernia 5Jonathan F. Finks

Chapter 3Incarcerated/Strangulated Inguinal Hernia 10Matthew W. Ralls Justin B. Dimick

Chapter 4Ventral Incisional Hernias 16Vivian M. Sanchez Kamal M.F. Itani

Chapter 5Complex Abdominal Wall Reconstruction 20Michael J. Rosen

Chapter 6Enterocutaneous Fistula 23Eric J. Culbertson Michael G. Franz

Chapter 7Infected Ventral Hernia Mesh 29Gregory Ara Dumanian

Chapter 8Postoperative Dehiscence 35Angela M. Ingraham Avery B. Nathens

Chapter 9Splenectomy for Hematologic Disease 38John F. Sweeney

Chapter 10Acute Appendicitis 43Sarah E. Greer Samuel R.G. Finlayson

Chapter 11Perforated Appendicitis 46Terry ShihMark R. HemmilaJustin B. Dimick

Chapter 12Gynecologic Causes of Lower Abdominal Pain 52Charles S. Dietrich III Bradford P. Whitcomb

GastrointestinalChapter 13Paraesophageal Hernia 58Thadeus Trus

Chapter 14Gastroesophageal Refl ux Disease 62Jonathan F. Finks

Chapter 15Gastric Cancer 66Srinivas KavuturuJussuf T. Kaifi Kevin F. Staveley-O’carroll

Chapter 16Bleeding Gastric Ulcer 72Danielle Fritze Michael Mulholland

Chapter 17Bleeding Duodenal Ulcer 77Wendy L. Wahl

Chapter 18Perforated Duodenal Ulcer 81Constance W. Lee George A. Sarosi Jr.

Chapter 19Small Bowel Obstruction 89Sara E. ClarkLillian G. Dawes

xxvii

Dimick_FM.indd xxviiDimick_FM.indd xxvii 3/30/2012 4:47:15 PM3/30/2012 4:47:15 PM

Page 28: Clinical Scenarios in Surgery - Booksca.ca...Clinical Scenarios in Surgery DECISION MAKING AND OPERATIVE TECHNIQUE Editors Justin B. Dimick, MD, MPH Assistant Professor of Surgery

xxviii Contents

Chapter 20Morbid Obesity 94John Morton

Chapter 21Gastrointestinal Stromal Tumor 97John B. Ammori Ronald P. Dematteo

Chapter 22Symptomatic Cholelithiasis in Pregnancy 102Vance L. Smith Paul M. Maggio

HepatobiliaryChapter 23Acute Cholecystitis 106Danielle Fritze Justin B. Dimick

Chapter 24Bile Duct Injury 113Christopher J. Sonnenday

Chapter 25Cholangitis 120William C. Beck Benjamin K. Poulose

Chapter 26Severe Acute Pancreatitis 125Marisa CevascoStanley W. AshleyAmy L. Rezak

Chapter 27Incidental Liver Mass 129Shawn J. Pelletier

Chapter 28Liver Mass in Chronic Liver Disease 134Christopher J. Sonnenday

Chapter 29Metastatic Colorectal Cancer 139Peter D. Peng Timothy M. Pawlik

Chapter 30Obstructive Jaundice 145Timothy L. Frankel Christopher J. Sonnenday

Chapter 31Incidental Pancreatic Cyst 152Joshua A. Waters C. Max Schmidt

Chapter 32Refractory Pain From Chronic Pancreatitis 158Sajid A. Khan Jeffrey B. Matthews

Chapter 33Symptomatic Pancreatic Pseudocyst 163Michael G. House

Chapter 34Cholecystoduodenal Fistula 166Paul Park Ali F. Mallat

ColorectalChapter 35Lower Gastrointestinal Bleeding 170Scott E. Regenbogen

Chapter 36Splenic Flexure Colon Cancer 175Daniel Albo

Chapter 37Anastomotic Leak After Colectomy 180Neil Hyman

Chapter 38Large Bowel Obstruction from Colon Cancer 185Noelle L. Bertelson David A. Etzioni

Chapter 39Colonic Vovulus 190Arden M. Morris

Dimick_FM.indd xxviiiDimick_FM.indd xxviii 3/30/2012 4:47:15 PM3/30/2012 4:47:15 PM

Page 29: Clinical Scenarios in Surgery - Booksca.ca...Clinical Scenarios in Surgery DECISION MAKING AND OPERATIVE TECHNIQUE Editors Justin B. Dimick, MD, MPH Assistant Professor of Surgery

Contents xxix

BreastChapter 50Palpable Breast Mass 239Travis E. GrotzSandhya PruthiJames W. Jakub

Chapter 51Suspicious Mammographic Abnormality 245Catherine E. Pesce Lisa K. Jacobs

Chapter 52Ductal Carcinoma In Situ 249Jessica M. Bensenhaver Tara M. Breslin

Chapter 53Lobular Carcinoma In Situ 255Lisa A. Newman

Chapter 54Advanced Breast Cancer 259Steven Chen Erin Brown

Chapter 55Infl ammatory Breast Cancer 263Walter P. Weber Monica Morrow

Chapter 56Breast Cancer During Pregnancy 267Jennifer E. Joh Marie Catherine Lee

Chapter 57Breast Reconstruction 272Jennifer F. Waljee Amy K. Alderman

EndocrineChapter 58Palpable Thyroid Nodule 283Haggi Mazeh Rebecca S. Sippel

Chapter 59Papillary Thyroid Carcinoma 287Gerard M. Doherty

Chapter 40Complicated Diverticulitis 195Sean T. Martin Jon D. Vogel

Chapter 41Ischemic Colitis 200Muneera R. Kapadia Ann C. Lowry

Chapter 42Medically Refractory Ulcerative Colitis 204Samantha Hendren

Chapter 43Crohn’s Disease with Small Bowel Stricture 208Hueylan Chern Emily Finlayson

Chapter 44Fulminant Clostridium diffi cile Colitis 212Natasha S. Becker Samir S. Awad

Chapter 45Appendiceal Carcinoid Tumor 216Wolfgang B. GaertnerElizabeth C. WickGenevieve Melton-Meaux

Chapter 46Rectal Cancer 220Hyaehwan Kim Martin R. Weiser

Chapter 47Anal Carcinoma 226A.V. Hayman A.L. Halverson

Chapter 48Perianal Abscess 229Richard E. Burney

Chapter 49Thrombosed Hemorrhoids 235Richard E. Burney

Dimick_FM.indd xxixDimick_FM.indd xxix 3/30/2012 4:47:15 PM3/30/2012 4:47:15 PM

Page 30: Clinical Scenarios in Surgery - Booksca.ca...Clinical Scenarios in Surgery DECISION MAKING AND OPERATIVE TECHNIQUE Editors Justin B. Dimick, MD, MPH Assistant Professor of Surgery

xxx Contents

ThoracicChapter 70Esophageal Cancer 348Mark B. Orringer

Chapter 71Esophageal Perforation 359Alykhan S. Nagji Christine L. Lau

Chapter 72Achalasia 363Tyler Grenda Jules Lin

Chapter 73Solitary Pulmonary Nodule 368James HarrisAlicia HulbertMalcolm V. Brock

Chapter 74Spontaneous Pneumothorax 374Pierre Theodore

VascularChapter 75Pulsatile Abdominal Mass 378Paul D. Dimusto Gilbert R. Upchurch Jr.

Chapter 76Ruptured Abdominal Aortic Aneurysm 383Adriana LaserGuillermo A. EscobarGilbert R. Upchurch Jr.

Chapter 77Lifestyle-Limiting Claudication 390Edouard Aboian Philip P. Goodney

Chapter 78Tissue Loss Due to Arterial Insuffi ciency 398William P. Robinson III

Chapter 79Acute Limb Ischemia 406Peter K. Henke John W. Rectenwald

Chapter 60Medullary Thyroid Cancer 293Barbara Zarebczan Herbert Chen

Chapter 61Primary Hyperparathyroidism 297Leslie S. Wu Julie Ann Sosa

Chapter 62Persistent Hyperparathyroidism 305James T. Broome

Chapter 63Incidental Adrenal Mass 312Brian D. Saunders Melissa M. Boltz

Chapter 64Adrenal Cancer 317David T. Hughes Paul G. Gauger

Chapter 65Cortisol-secreting Adrenal Tumor 321Hari R. Kumar Judiann Miskulin

Chapter 66Primary Hyperaldosteronism 325Barbra S. Miller

Chapter 67Pheochromocytoma 329Alexis D. Smith Douglas J. Turner

Chapter 68Pancreatic Neuroendocrine Tumors 334Stacey A. Milan Charles J. Yeo

Chapter 69Gastrinoma 340Geoffrey W. Krampitz Jeffrey A. Norton

Dimick_FM.indd xxxDimick_FM.indd xxx 3/30/2012 4:47:15 PM3/30/2012 4:47:15 PM

Page 31: Clinical Scenarios in Surgery - Booksca.ca...Clinical Scenarios in Surgery DECISION MAKING AND OPERATIVE TECHNIQUE Editors Justin B. Dimick, MD, MPH Assistant Professor of Surgery

Contents xxxi

Chapter 89Neuroblastoma 455Erika Newman Ihab Halaweish

Chapter 90Palpable Abdominal Mass in a Toddler 459Douglas C. Barnhart

Chapter 91Hepatoblastoma 463Terry L. Buchmiller

Chapter 92Intussusception 467Sabina Siddiqui James D. Geiger

Chapter 93Necrotizing Enterocolitis 471Richard Herman Daniel H. Teitelbaum

Chapter 94Rectal Bleeding in a Young Child 477Gavin A. Falk Oliver S. Soldes

Chapter 95Omphalocele 481Emily M. Fontenot Sean E. Mclean

Chapter 96Gastroschisis 486Samir K. Gadepalli James D. Geiger

Chapter 97Tracheoesophageal Fistula 491Steven W. Bruch

Skin and Soft TissueChapter 98Melanoma 498Sebastian G. De La FuenteTimothy W. MccardleVernon K. Sondak

Chapter 80Asymptomatic Carotid Stenosis 411Paul D. DimustoJohn E. RectenwaldGilbert R. Upchurch Jr.

Chapter 81Symptomatic Carotid Stenosis 417Sarah M. Weakley Peter H. Lin

Chapter 82Diabetic Foot Infection 421Jeffrey Kalish Allen Hamdan

Chapter 83Acute Mesenteric Ischemia 426Babak J. Orandi James H. Black III

Chapter 84Chronic Mesenteric Ischemia 431Maureen K. SheehanMatthew J. Sideman Kevin E. Taubman

Chapter 85Deep Venous Thrombosis 437Justin HurieThomas W. Wakefi eld

Chapter 86Need for Hemodialysis Access 441Alexis D. Jacob Thomas S. Huber

PediatricChapter 87Emesis in an Infant 446Erica R. Gross Robert A. Cowles

Chapter 88Malrotation and Midgut Volvulus 450Adam S. Brinkman Dennis P. Lund

Dimick_FM.indd xxxiDimick_FM.indd xxxi 3/30/2012 4:47:15 PM3/30/2012 4:47:15 PM

Page 32: Clinical Scenarios in Surgery - Booksca.ca...Clinical Scenarios in Surgery DECISION MAKING AND OPERATIVE TECHNIQUE Editors Justin B. Dimick, MD, MPH Assistant Professor of Surgery

xxxii Contents

Chapter 99Melanoma Presenting with Regional Lymph Node Involvement 505Michael S. Sabel

Chapter 100Merkel Cell Carcinoma 508Michael S. Sabel

Chapter 101Nonmelanoma Skin Cancer 511Anastasia Dimick

Chapter 102Necrotizing Soft Tissue Infections 514Michelle K. Mcnutt Lillian S. Kao

Chapter 103Extremity Mass (Sarcoma) 518Timothy L. Frankel Alfred E. Chang

Chapter 104Retroperitoneal Sarcoma 524Chandu Vemuri Sandra L. Wong

TraumaChapter 105Penetrating Chest Injury 529Albert Chi Adil H. Haider

Chapter 106Stab Wound to the Neck 535Gina M.S. Howell Jason L. Sperry

Chapter 107Burns 540Jeffrey S. Guy

Chapter 108Blunt Abdominal Trauma from Motor Vehicle Crash 546Carla Kohoyda-Inglis Stewart C. Wang

Chapter 109Duodenal Injury 550Filip Bednar Mark R. Hemmila

Chapter 110Pelvic Fracture 555Avi Bhavaraju Oliver L. Gunter

Critical CareChapter 111Airway Emergency 560Derek T. Woodrum David W. Healy

Chapter 112Acute Renal Failure 566April E. Mendoza Anthony G. Charles

Chapter 113Adrenal Insuffi ciency 571Steven R. Allen Heidi L. Frankel

Chapter 114Acute Respiratory Distress Syndrome (ARDS) 574Pauline K. ParkKrishnan RaghavendranLena M. Napolitano

Chapter 115Ventilator-associated Pneumonia 581Krishnan Raghavendran

Chapter 116Septic Shock 584Pamela A. Lipsett

Chapter 117Abdominal Compartment Syndrome 589Rebecca Plevin Heather L. Evans

Chapter 118Nutritional Support in the Critically Ill Surgery Patient 594Kyle J. Van Arendonk Elliott R. Haut

Dimick_FM.indd xxxiiDimick_FM.indd xxxii 3/30/2012 4:47:15 PM3/30/2012 4:47:15 PM

Page 33: Clinical Scenarios in Surgery - Booksca.ca...Clinical Scenarios in Surgery DECISION MAKING AND OPERATIVE TECHNIQUE Editors Justin B. Dimick, MD, MPH Assistant Professor of Surgery

Contents xxxiii

Head and NeckChapter 122Melanoma of the Head and Neck 613Andrew KroekerAndrew ShumanErin Mckean

Chapter 123Head and Neck Cancer 618Matthew Spector Erin Mckean

Index 623

TransplantChapter 119Acute Liver Failure 600Bernard J. Dubray Christopher D. Anderson

Chapter 120Variceal Bleeding and Portal Hypertension 605Brendan J. Boland Andrew S. Klein

Chapter 121End-Stage Renal Disease (Renal Transplantation) 609Leigh Anne Redhage Derek Moore

Dimick_FM.indd xxxiiiDimick_FM.indd xxxiii 3/30/2012 4:47:15 PM3/30/2012 4:47:15 PM

Page 34: Clinical Scenarios in Surgery - Booksca.ca...Clinical Scenarios in Surgery DECISION MAKING AND OPERATIVE TECHNIQUE Editors Justin B. Dimick, MD, MPH Assistant Professor of Surgery

Dimick_FM.indd xxxivDimick_FM.indd xxxiv 3/30/2012 4:47:15 PM3/30/2012 4:47:15 PM

This page intentionally left blank.

Page 35: Clinical Scenarios in Surgery - Booksca.ca...Clinical Scenarios in Surgery DECISION MAKING AND OPERATIVE TECHNIQUE Editors Justin B. Dimick, MD, MPH Assistant Professor of Surgery

1

1 Symptomatic Primary Inguinal HerniaEVANGELOS MESSARIS

the symptomatic side) to rule out bilateral inguinal hernias. No laboratory studies can help with the diag-nosis of an inguinal hernia.

Rarely the use of imaging studies is helpful in mov-ing from the differential diagnosis to a single work-ing diagnosis. Imaging studies are mostly used in obese patients where physical exam has limitations (Figure 1). An ultrasound can demonstrate or rule out enlarged inguinal nodes, hydroceles, testicular tor-sion, varicocele, spermatocele, epididymal cyst, and testicular tumors. Furthermore, an experienced ultra-sonographer can demonstrate an inguinal hernia sac and identify its contents. Computed tomography is mostly used on cases of very large inguinal hernias, to depict the contents of the sac and to identify aberrant anatomy in the inguinal canal (Figure 2).

Diagnosis and TreatmentAscertaining whether patients have symptoms from their hernia is important for decision making. For truly asymptomatic hernias, a watchful waiting strategy can be followed. Younger patients are almost always symp-tomatic because they are invariably active. However, older patients who are not physically active may not be bothered by their hernia and repair can be deferred indefi nitely.

Inguinal hernias can present with many different symptoms. A reducible hernia will often present with groin discomfort that is exacerbated with activity. Patients with incarceration or strangulation will pres-ent with more severe pain and, potentially overly-ing skin erythema. The treatment of all symptomatic inguinal hernias is surgical repair. The goals of the repair are to relieve the symptoms and prevent any future incarceration or strangulation of the hernia. The timing for symptomatic hernia repairs depends on whether the hernia is reducible, incarcerated, or

Differential DiagnosisGroin discomfort usually is associated with an ingui-nal or femoral hernia or a process involving the sper-matic cord or round ligament structures. Although, inguinal hernias are common, there are other medical conditions that can have similar presentation. Femoral hernias, enlarged inguinal nodes, hydroceles, testicu-lar torsion, epididymitis, varicocele, spermatocele, epi-didymal cyst, and testicular tumors are less frequent but should be included in the differential diagnosis of a patient presenting with a symptomatic groin mass or groin discomfort.

WorkupThe patient undergoes more extensive physical exam of his abdomen, in the standing and supine position, demonstrating a reducible inguinal mass at the level of the external ring of the inguinal canal with minimal overlying tenderness, suggestive of a right inguinal hernia.

The diagnosis of an inguinal hernia is based on physi-cal examination. Reported sensitivity and specifi city of physical examination for the diagnosis of inguinal her-nia are 75% and 96%, respectively. In males, the index fi nger of the examiner should invaginate the scrotum in an attempt to fi nd the external opening of the ingui-nal canal. The patient should then be asked to cough or perform a Valsalva maneuver. The examiner should then feel the hernia sac with all its contents at the tip of his index fi nger. Similarly, in female patients the exam-iner can feel for the hernia sac by palpating the ingui-nal area just laterally of the pubic tubercle. It should be noted that the exam is performed above the inguinal ligament, because if the protruding mass is below the inguinal ligament, then it is a femoral hernia. This dis-tinction is not often easy, especially in obese patients. In all cases both sides should be examined (not only

Presentation

A 55-year-old male patient with a history of hypertension and diabetes presents with right groin discomfort. He reports having right groin discomfort for the last 3 months. He also noticed a bulge in his right groin several months ago. He has no fever, chills, nausea, vomiting or dysuria. His vitals are normal. On exam it is noted that he has a mass in the right groin that extends into his scrotum. The mass is reducible, but it imme-diately recurs after reduction.

Dimick_Chap01.indd 1Dimick_Chap01.indd 1 3/15/2012 7:24:45 PM3/15/2012 7:24:45 PM

Page 36: Clinical Scenarios in Surgery - Booksca.ca...Clinical Scenarios in Surgery DECISION MAKING AND OPERATIVE TECHNIQUE Editors Justin B. Dimick, MD, MPH Assistant Professor of Surgery

2 Clinical Scenarios in Surgery

strangulated. Reducible hernias can be repaired in an elective outpatient fashion, incarcerated hernias warrant urgent repair within 12 hours of presentation, and strangulated hernias need to go to the operating room emergently, since the viability of an organ in the hernia sac is compromised.

Surgical ApproachThe surgical approach for a symptomatic inguinal hernia could be open or laparoscopic, with local, spinal, or general anesthesia. In the open proce-dures the repair can be suture based (Bassini, McVay, Shouldice) or using mesh (e.g., Lichtenstein). Mesh is also used in all the laparoscopic cases that can be further divided in total extraperitoneal (TEP) and transabdominal preperitoneal (TAPP), depending on whether the peritoneal cavity is used for access to the inguinal region or not. Although many suggest using open repair for unilateral primary hernias and laparoscopic repair for bilateral and recurrent ingui-nal hernias, surgeon’s experience should guide the choice of repair. Laparoscopic inguinal hernia repair has a steep learning curve, and most experts suggest 100 to 250 cases are necessary to develop profi ciency. For surgeons who are not profi cient at laparoscopic herniography, open mesh repair is the best choice, even for recurrences and bilateral repairs.

Regardless of the technique employed, the main goal of surgical therapy is a tension-free repair of the defect to decrease the recurrence rate. All elective and the majority of the emergent repairs, except those where bowel is compromised and a bowel resection is performed, achieve this goal by placing mesh over

the defect, or in the case of the laparoscopic approach, behind the defect. In contaminated cases, a suture-based technique (Bassini, McVay, or Shouldice) or bio-logic mesh can be used. However, these patients will have a higher recurrence rate.

Preoperative CareAll patients are placed in a supine position on the oper-ating table. Patients should have thigh-length sequen-tial compression devices and in our practice we give 5,000 units of unfractionated heparin subcutaneously if they are older than 40 years. Administration of a fi rst-generation cephalosporin intravenously within 1 hour prior to incision is recommended, especially in cases where mesh is going to be used. Skin prepara-tion should be done with chlorhexidine and should include the scrotum, in case manipulation is needed for the hernia sac reduction or to facilitate the return of the testicle into its proper location.

Local anesthesia can be given either as a nerve block of the ilioinguinal and iliohypogastric nerves or as direct infi ltration into the incision site, always in com-bination with some conscious sedation. Alternatively, spinal or general anesthesia can be used.

All patients should void prior to the procedure, oth-erwise intraoperative bladder decompression with a bladder catheter is advised.

Open Inguinal Hernia RepairLichtenstein open, tension-free hernioplasty is con-sidered the “gold standard” for open hernia repair (Table 1). The skin incision is placed over the inguinal

FIGURE 1 • Axial cut of a CT demonstrating a moderate-size right inguinal hernia with omentum in the hernia sac in an obese patient where physical exam fi ndings would be limited.

FIGURE 2 • Axial cut of a CT, demonstrating a left inguinal hernia with sigmoid colon in the hernia sac.

Dimick_Chap01.indd 2Dimick_Chap01.indd 2 3/15/2012 7:24:46 PM3/15/2012 7:24:46 PM

Page 37: Clinical Scenarios in Surgery - Booksca.ca...Clinical Scenarios in Surgery DECISION MAKING AND OPERATIVE TECHNIQUE Editors Justin B. Dimick, MD, MPH Assistant Professor of Surgery

Symptomatic Primary Inguinal Hernia 3

canal and angled only slightly cephalad as it progresses laterally. The major anatomical landmark is exposure over the pubic tubercl, medially. The incision is carried down to the abdominal wall fascia that consists of the external oblique aponeurosis to expose the external inguinal ring. The aponeurosis is incised in the direc-tion of its fi bers. The cord structures are dissected from the cremasteric muscle and transversalis fascia fi bers and retracted off the inguinal canal fl oor. The cord is explored for an indirect hernia sac or cord lipoma. All hernia sacs and cord lipomas are transected at the level of the internal ring. An appropriate size polypropylene mesh is secured to the shelving edge of the inguinal ligament from the pubic tubercle to past the insertion of the arch of the internal oblique to Poupart’s liga-ment using running or interrupted 2-0 Prolene suture. Similarly, the upper edge of the mesh is sutured to the rectus sheath and internal oblique muscle. The internal ring is reconstructed by suturing the two leaves of the mesh together lateral to the cord. The spermatic cord is returned to its original position and the aponeuro-sis of the external oblique is reapproximated using 2-0 absorbable suture in a running fashion, avoiding inju-ries of the ilioinguinal nerve.

Laparoscopic Inguinal Hernia RepairThe TEP repair of inguinal hernias was developed out of concern for possible complications related to

intra-abdominal access required for transabdominal approach (Table 2). In detail, the skin incision is made at the inferior aspect of the umbilicus and the anterior rectus sheath is incised lateral to the midline. Blunt dissection is used to sweep the rectus muscle laterally from the midline to expose the posterior rectus sheath fascia. A dissecting balloon is placed in the space between the rectus muscle anteriorly and the poste-rior fascia, and directed down to the pubis. Under direct visualization, the dissector is infl ated. The bal-loon is then replaced by a standard blunt port and the previously created extraperitoneal space is insuf-fl ated with CO2 to reach 12 mm Hg. Two 5-mm trocars are placed in the lower midline. After identifi cation of the inferior epigastric vessels superiorly, Cooper’s ligament medially, and the ileopubic tract laterally, the hernia sac is reduced, paying particular attention to completely detach the sac off the cord structures. A preformed or custom-made polyester mesh can be used for the repair. The mesh is positioned from a medial to lateral direction under the cord structures paying particular attention to cover the internal ring both laterally and superiorly, while its medial aspect is tucked below the Cooper’s ligament. When the

TABLE 1. Key Steps to Open Lichtenstein Tension-free Hernioplasty

1. The skin incision is placed over the inguinal canal for exposure of the pubic tubercle.

2. The cord structures are dissected from the crem-asteric muscle and transversalis fascia fi bers and retracted off the inguinal canal fl oor.

3. The cord is explored for an indirect hernia sac or cord lipoma.

4. Polypropylene mesh is secured inferiorly to the shelv-ing edge of the inguinal ligament and superiorly to the rectus sheath and internal oblique muscle.

5. The internal ring is reconstructed by suturing the two leaves of the mesh together.

6. The spermatic cord is returned to its original posi-tion and the aponeurosis of the external oblique is reapproximated.

7. Check that testicles are still in the proper anatomical position in the scrotum.

Potential Pitfalls

• The pubic tubercle must be completely covered with mesh; if not there is higher risk for recurrence.

• Avoid entrapment of ilioinguinal, iliohypogastric, or geni-tofemoral nerves.

• Mesh fi xation should be tension free.• Confi rm that spermatic vessels are intact and that tes-

ticles are in proper position at the end of the procedure.

TABLE 2. Key Steps to Laparoscopic Totally Extraperitoneal Repair of Inguinal Hernia

1. Enter rectus sheath through dissection from a infraum-bilical skin incision.

2. A bluntly dissecting balloon is placed in the space between the rectus muscle anteriorly and the poste-rior fascia, and directed down to the pubis.

3. Two 5-mm trocars are placed in the lower midline between the rectus muscles.

4. Proper identifi cation of critical anatomical landmarks is essential (the inferior epigastric vessels superiorly, Cooper’s ligament medially, and the ileopubic tract laterally).

5. Hernia sac is reduced and separated off the cord structures.

6. A preformed or custom-made polyester mesh is posi-tioned from a medial to lateral direction under the cord structures paying particular attention to cover the inter-nal ring both laterally and superiorly, while its medial aspect is tucked below the Cooper’s ligament.

7. Mesh fi xation is not needed.

Potential Pitfalls

• Blunt dissection in the wrong plane or previous surgery in the pelvic or inguinal region may provide poor visual-ization of the landmark structures.

• Injury to the inferior epigastric vessels should be avoided.• Incomplete hernia sac reduction and dissection off the

cord structures may lead to incomplete repair and early recurrence.

• Nerve injuries are more common in laparoscopic repairs.

Dimick_Chap01.indd 3Dimick_Chap01.indd 3 3/15/2012 7:24:46 PM3/15/2012 7:24:46 PM

Page 38: Clinical Scenarios in Surgery - Booksca.ca...Clinical Scenarios in Surgery DECISION MAKING AND OPERATIVE TECHNIQUE Editors Justin B. Dimick, MD, MPH Assistant Professor of Surgery

4 Clinical Scenarios in Surgery

mesh is correctly positioned, it can be fi xated using tacks, staples, fi brin glue, or just be left in place with-out any fi xation.

Special Intraoperative ConsiderationsIn all inguinal hernia repair cases, all types and all approaches, the major key point for a successful opera-tion is knowing the anatomy of the inguinal canal (Tables 1 and 2).

For open repairs, attention should be paid to the dissection and preservation of the ilioinguinal and iliohypogastric nerve. Nerve entrapment can cause signifi cant neuralgia in the postoperative period. If during the procedure a nerve is injured, then complete transection of the nerve is advised.

During laparoscopic repairs, the dissection in the groin area will cause some lacerations to the peri-toneum and the peritoneal cavity contents maybe encountered. Each defect of the peritoneum should be closed using an endo-loop ligature (2-0 vicryl), and if the peritoneal cavity is insuffl ated with CO2, then it can be decompressed using a Veress needle.

Intraoperative complications include femoral ves-sel or inferior epigastric vessel injuries, bladder or testicular injuries, and vas deferens injury or nerve injury.

Postoperative ManagementFor elective cases or cases with omental incarcera-tion, the patient usually can be discharged within 3 to 4 hours postoperatively. The patient should void without any problems and have adequate pain control before being discharged. Urinary retention is frequent after inguinal surgery and it is associated with the use of narcotics, the type of surgery, and the amount of intravenous fl uids administered to the patients.

For urgent or emergent cases if no bowel was affected usually 24 hours of observation are adequate before discharge. In cases where bowel was found strangu-lated and bowel resection was done, the patients are usually followed in the hospital for 2 to 3 days.

Follow-up in all cases usually is scheduled 3 to 4 weeks postoperatively to check the wound healing (rule out any wound infections—rare <1%, or seromas or hematomas). Routine examination should rule out early recurrence and any neuralgia from nerve injury or entrapment. Most patients are able to return to work within 2 weeks from surgery, and even earlier if per-formed laparoscopically. No heavy weight lifting is advisable up to 3 months from the operation.

TAKE HOME POINTS

• Inguinal hernias are common, comprising three-fourths of all abdominal wall defects. Lifetime risk for developing an inguinal hernia is 15% for males and 5% for females.

• All symptomatic inguinal hernias need to be surgi-cally repaired to relieve symptoms and prevent any future incarceration or strangulation of the hernia.

• There are several described procedures for inguinal hernia repair and they can be open or laparoscopic.

• Regardless of the technique employed, the main goal of surgical therapy is a tension-free repair of the defect to decrease the recurrence rate.

• Seromas, neuralgia, and recurrence are some of the most frequent postoperative complications.

SUGGESTED READINGS

Amato B, Moja L, Panico S, et al. Shouldice technique versus other open techniques for inguinal hernia repair. Cochrane Database Syst Rev. 2009;(4):CD001543.

Langeveld HR, van’t Riet M, Weidema WF, et al. Total extraperitoneal inguinal hernia repair compared with Lichtenstein (the LEVEL-Trial): a randomized controlled trial. Ann Surg. 2010;251(5):819–824.

Messaris E, Nicastri G, Dudrick SJ. Total extraperitoneal lap-aroscopic inguinal hernia repair without mesh fi xation: prospective study with 1-year follow-up results. Arch Surg. 2010;145(4):334–338.

Neumayer L, Giobbie-Hurder A, Jonasson O, et al.; Veterans Affairs Cooperative Studies Program 456 Investigators. Open mesh versus laparoscopic mesh repair of inguinal hernia. N Engl J Med. 2004;350(18):1819–1827.

Nordin P, Zetterström H, Gunnarsson U, et al. Local, regional, or general anaesthesia in groin hernia repair: multicentre randomised trial. Lancet. 2003;362(9387):853–858.

Case Conclusion

The patient underwent a successful laparoscopic right inguinal repair with mesh and was dis-charged 4 hours postoperatively. He returned to the offi ce in 3 weeks with well-healed port sites and was pain free. During his routine postopera-tive appointment, the patient reported feeling a bulge in the right groin that was similar to the hernia that he had before. Exam did not reveal a recurrence and an ultrasound demonstrated a seroma at the repair site. No intervention was per-formed and the patient was seen 3 months postop-eratively and the seroma was completely resolved.

Dimick_Chap01.indd 4Dimick_Chap01.indd 4 3/15/2012 7:24:46 PM3/15/2012 7:24:46 PM

Page 39: Clinical Scenarios in Surgery - Booksca.ca...Clinical Scenarios in Surgery DECISION MAKING AND OPERATIVE TECHNIQUE Editors Justin B. Dimick, MD, MPH Assistant Professor of Surgery

5

2

space, while avoiding the thick lower abdominal wall pannus. The TAPP repair is also useful in cases of large scrotal hernias, as these can be more easily reduced from the peritoneal cavity than from the preperitoneal space. The transabdominal approach also allows for assess-ment of bowel viability in cases of strangulated hernias. Finally, conversion to TAPP repair may also be required during an attempted TEPP repair if, for example, the peritoneum is violated while attempting to develop the preperitoneal space with a balloon dissector. This latter scenario often occurs in patients with lower abdominal incisions (e.g., Pfannenstiel).

Surgical ApproachIn essence, the TAPP procedure for inguinal hernia repair involves entry into the preperitoneal space by incision of the lower abdominal wall peritoneum from inside the peritoneal cavity (Table 1). Once in the pre-peritoneal space, the hernia sac is dissected free from the cord structures and reduced from within the deep inguinal ring (indirect hernia), Hesselbach’s triangle (direct hernia), and/or the femoral space (femoral hernia). Once the hernia contents have been reduced, the peritoneum is dissected well off of the cord struc-tures to make room for placement of the mesh. Mesh is then placed such that it adequately covers the direct, indirect, and femoral spaces. The peritoneum is then secured up to the abdominal wall to cover the mesh.

The procedure is performed under general anesthe-sia with the patient supine, both arms tucked to the side, in slight Trendelenburg position. A Foley catheter is inserted to decompress the bladder. Access to the peritoneum is obtained using a closed (Veress) or an open (Hasson) technique, and pneumoperitoneum is established. The surgeon stands on the side opposite the hernia, with the assistant on the ipsilateral side (Figure 1). An 11-mm trocar is placed above the umbi-licus in the midline for placement of the laparoscope

Differential DiagnosisThe leading diagnosis based on these symptoms is a recurrent right inguinal hernia. Other considerations would include lymphadenopathy; soft tissue mass, such as a lipoma or a sarcoma; and hematoma related to trauma.

WorkupTo evaluate for recurrent hernia, the best imaging study is a CT of the abdomen and pelvis, with at least oral contrast. Two sets of images should be obtained: the fi rst using a standard technique and the second with the patient performing a Valsalva maneuver. This test will allow for better identifi cation of hernia contents in the inguinal canal.

Diagnosis and TreatmentIn this case, cross-sectional imaging demonstrated a recurrent right inguinal hernia containing nonob-structed loops of small bowel. The left inguinal canal was normal in appearance. Given the symptomatic nature of this hernia, repair is warranted. There are several options for surgical management. An anterior approach would be very diffi cult and unlikely to pro-duce durable results, given the patient’s body habitus and the presence of previously placed mesh. A preperi-toneal approach is preferred in this case because the repair would be done in an unviolated tissue plane. Furthermore, this technique results in coverage of the direct, indirect, and femoral spaces. This could be done using an open preperitoneal technique but would be diffi cult given the patient’s obesity and large pannus. Similarly, a total extraperitoneal (TEPP) approach would also be hindered by a thick abdominal wall and limited working space due to adipose tissue in the preperitoneal space. In this case, I believe the best technique would be a transabdominal preperitoneal (TAPP) approach. The transabdominal route allows access to the preperitoneal

Presentation

A 50-year-old obese man with a large pannus is referred for evaluation of a recurrent right inguinal bulge occurring 5 years following open mesh repair of a right inguinal hernia. He has noticed the bulge for the last several months. Although reducible, the patient has noted increasing discomfort associated with the bulge over the last few weeks. He denies any obstructive symptoms and has had no symptoms on the left side. Physical exam demonstrates some fullness in the right groin, but the exam is limited by the patient’s body habitus.

Recurrent Inguinal HerniaJONATHAN F. FINKS

Dimick_Chap02.indd 5Dimick_Chap02.indd 5 3/15/2012 7:26:09 PM3/15/2012 7:26:09 PM

Page 40: Clinical Scenarios in Surgery - Booksca.ca...Clinical Scenarios in Surgery DECISION MAKING AND OPERATIVE TECHNIQUE Editors Justin B. Dimick, MD, MPH Assistant Professor of Surgery

6 Clinical Scenarios in Surgery

and later insertion of the mesh into the peritoneal cav-ity. Many surgeons prefer to work through ports on both sides of the midline so as to effect proper trian-gulation (Figure 1). However, in the obese individual, the surgeons’ working ports (both 5-mm ports) should both be on the side contralateral to the hernia, usually on either side of the midclavicular line and below the level of the umbilicus. In some cases, an additional 5-mm assistant’s port may be placed on the ipsilat-eral side, at the midclavicular line above the level of the umbilicus. In the case of bilateral inguinal hernia repair, the working trocars are generally placed at or above the level of the umbilicus. A 10-mm 30° laparo-scope is employed, although some surgeons prefer a 0° laparoscope in nonobese patients.

The procedure begins with an inspection of the lower abdominal wall on both sides. Figure 2 shows the anatomy and landmarks in the right lower abdo-men. The median umbilical ligaments and epigastric

vessels should be identifi ed on either side of the blad-der. Any obvious hernia defects should be identifi ed, although some of these may not be apparent until the peritoneum is taken down. Indirect hernias are located lateral to the inferior epigastric vessels. Direct hernias occur through Hesselbach’s triangle, bordered later-ally by the inferior epigastric vessels, medially by lat-eral edge of the rectus muscle, and inferiorly by the inguinal ligament. Femoral hernias occur through the femoral space, bordered laterally by the femoral vein, posteriorly by Cooper’s ligament, and anteriorly by the inguinal ligament.

The preperitoneal space is then developed begin-ning with an incision in the peritoneum using elec-trocautery. The incision begins vertically along the ipsilateral median umbilical ligament down to its root. The incision is carried transversely above the level of the hernia defects, across to the anterior superior iliac spine (Figure 3). In cases of a bilateral inguinal hernia,

10 mm

5 mm

First assistant

FIGURE 1 • Operating room setup and trocar placement for a TAPP hernia repair. (From Soper, Swanstrom, Eubanks. Mastery of Endoscopic and Laparoscopic Surgery. 3rd ed. Lippincott Williams and Wilkins, 2009, Figure 53-13.)

Dimick_Chap02.indd 6Dimick_Chap02.indd 6 3/15/2012 7:26:09 PM3/15/2012 7:26:09 PM

Page 41: Clinical Scenarios in Surgery - Booksca.ca...Clinical Scenarios in Surgery DECISION MAKING AND OPERATIVE TECHNIQUE Editors Justin B. Dimick, MD, MPH Assistant Professor of Surgery

Recurrent Inguinal Hernia 7

a mirror incision is made on the opposite side. Separate dissections and pieces of mesh are used to repair bilat-eral hernias. Blunt and sharp dissection with elec-trocautery is then used to develop the preperitoneal space, staying close to the peritoneum. This dissection begins lateral to the cord structures, in Bogros’ space, advances medially toward the retropubic space, and extends proximally to expose the femoral vessels, psoas muscle, and retroperitoneum (Figure 3). Medially, the bladder is carefully dissected off of the anterior abdom-inal wall, exposing the symphysis pubis and Cooper’s ligament. Care must be taken not to injure corona mor-tis, which refers to the venous connection between the inferior epigastric and obturator veins. This structure courses inferiorly along the lateral aspect of Cooper’s ligament and, because of its location on the pubic bone, can be diffi cult to control if lacerated or avulsed.

An assessment for femoral and direct hernia defects occurs during the medial dissection. Careful attention is paid to identify the critical structures: inferior epi-gastric vessels, Cooper’s ligament, and the femoral vein. Direct and femoral hernias may contain only pre-peritoneal fat or they may contain a hernia sac. It is not uncommon for direct hernias to contain the urinary bladder. The hernia contents are reduced with gentle

blunt dissection. With a direct hernia, there is usually a clear transition between the transversalis fascia and the hernia sac. These structures can often be separated by applying cephalad and posterior retraction of the sac and anterior and caudad retraction of the trans-versalis fascia. In the setting of a large direct defect, large seromas may develop. To help minimize the risk for seroma formation, the transversalis fascia may be reduced from within Hesselbach’s triangle and tacked to Cooper’s ligament. When reducing femoral hernias, care must be taken to carefully delineate between her-nia contents and the fat and lymphatic tissue intimately associated with the femoral vein. Injudicious dissec-tion can lead to injury to the femoral vein. The medial dissection may also reveal an obturator hernia, located posterior to Cooper’s ligament through the obturator foramen. These are also reduced by blunt dissection and may require an additional medially placed mesh to cover the defect.

An indirect hernia is identifi ed during the lateral dissection. The hernia sac is bluntly dissected away from the underlying spermatic cord structures, namely the vas deferens and the testicular vessels. The sac must be dissected free from the cord structures prior to reduction of the sac from within the deep inguinal

FIGURE 2 • Laparoscopic view and anatomy of right lower abdominal wall seen during TAPP hernia repair. (From Soper, Swanstrom, Eubanks. Mastery of Endoscopic and Laparoscopic Surgery. 3rd ed. Lippincott Williams and Wilkins, 2009, Figure 53-14.)

Dimick_Chap02.indd 7Dimick_Chap02.indd 7 3/15/2012 7:26:10 PM3/15/2012 7:26:10 PM

Page 42: Clinical Scenarios in Surgery - Booksca.ca...Clinical Scenarios in Surgery DECISION MAKING AND OPERATIVE TECHNIQUE Editors Justin B. Dimick, MD, MPH Assistant Professor of Surgery

8 Clinical Scenarios in Surgery

ring to avoid inadvertent laceration or transection of the vas deferens or testicular vessels. The hernia sac is then reduced by application of cephalad and posterior retraction on the hernia sac, with anterior and caudad retraction of the transversalis fascia. We do not employ cautery during this dissection, especially in the space lateral to the cord structures, to avoid injury to the gen-ital branch of the genitofemoral nerve, which courses anterior to the psoas muscle in the pelvis and passes through the inguinal canal along with the cord in the lateral bundle of the cremasteric fascia.

Care must be taken to ensure that the hernia sac remains free from the cord structures during this entire process, particularly in the setting of a large scrotal sac. If the peritoneal sac is very large and cannot be easily reduced, it may be transected, with the distal aspect allowed to retract into the scrotum. The proximal aspect of the sac must then be secured during reperito-nealization following the mesh repair to prevent bowel adhesions to the mesh. Transection of the sac is safe but may lead to development of a hydrocele in some cases. Preperitoneal fat within the deep inguinal ring (cord lipomas) should be completely reduced from that space in order to prevent the patient’s sensation of a persistent bulge following hernia repair.

Once the hernia sac has been reduced, the perito-neum is dissected off of the cord structures in a cepha-lad direction. Adequate parietalization of the cord is essential, as it prevents peritoneum from slipping

underneath the bottom edge of the mesh, which leads to lateral recurrences. Similarly, herniated preperito-neal fat must also be dissected well off of the cord so that it cannot slip beneath the mesh. This dissection continues cephalad to the level of the anterior superior iliac spine and laterally to the iliac wing, allowing for exposure of the psoas muscle. Medially, this continues to the transition to the urinary bladder, which is then itself dissected off of Cooper’s ligament and the pubis in order to clear a space for placement of the mesh. Gentle medial retraction on the bladder allows for bet-ter delineation between prevesicular fat and fat associ-ated with the femoral vein and helps reduce the risk of inadvertent injury to the vein.

Once hemostasis has been ensured, the next step involves placement of a large piece of nonabsorbable mesh. We employ an anatomically contoured, light-weight, woven polypropylene mesh that is 10 cm in height by 16 cm in width. The mesh must be large enough to cover the direct, indirect, and femoral spaces (myopectineal orifi ce) and the posterior aspect of Cooper’s ligament. In the case of bilateral hernias, two pieces of mesh are used. The mesh is rolled and inserted into the abdomen through the 10-mm port. It is inserted into the preperitoneal space and unrolled such that the inferior aspect is draped over the cord structures and psoas muscle laterally and Cooper’s ligament and pubic symphysis medially. The superior aspect of mesh then covers the anterior abdominal wall above the level of

FIGURE 3 • Peritoneal incision (solid line) and extent of dissection (dashed line) in a left-sided TAPP hernia repair. (From Soper, Swanstrom, Eubanks. Mastery of Endoscopic and Laparoscopic Surgery. 3rd ed. Lippincott Williams and Wilkins, 2009, Figure 53-1.)

Dimick_Chap02.indd 8Dimick_Chap02.indd 8 3/15/2012 7:26:11 PM3/15/2012 7:26:11 PM

Page 43: Clinical Scenarios in Surgery - Booksca.ca...Clinical Scenarios in Surgery DECISION MAKING AND OPERATIVE TECHNIQUE Editors Justin B. Dimick, MD, MPH Assistant Professor of Surgery

Recurrent Inguinal Hernia 9

the iliopubic tract, including the inferior epigastric ves-sels and the rectus muscle medially. We tack the mesh medially to Cooper’s ligament with a single 5-mm spi-ral tack to prevent the mesh from sliding and will tack to the rectus muscle in cases of a large direct hernia to prevent the mesh from herniating through the defect. We avoid any tack placement laterally to prevent injury to the ilioinguinal and iliohypogastric nerves.

Once the mesh has been placed, the peritoneum is closed. This is facilitated by reducing the pneumoperito-neum pressure as low as possible, while still permitting adequate visualization. The entire peritoneum must be secured and the mesh covered to prevent bowel adhe-sions to the mesh or incarceration of a bowel loop within the preperitoneal space. This can be accomplished using spiral tacks, suture, or a combination of these.

Special Intraoperative ConsiderationsIn general, it is easy to get disoriented during laparo-scopic inguinal hernia repairs, whether done as a TEPP or a TAPP procedure, and this can lead to disastrous consequences. In the setting of a large indirect hernia sac, particularly in an obese patient, it can be diffi cult to identify the cord structures and this can lead to

dissection in the deeper “triangle of doom” with inad-vertent injury to the femoral artery or vein. It is worth-while to periodically identify known landmarks, such as Cooper’s ligament and the symphysis pubis as well as the inferior epigastric vessels. Such periodic reori-enting is often very helpful in keeping the dissection in the proper plane. In the event of a femoral vein injury, conversion to open will most likely be required. First, however, the surgeon should increase the pneumo-peritoneum pressure to 25 mm of mercury or higher as necessary to help tamponade the bleeding. Direct pres-sure with a Raytec opened completely and inserted through the 10-mm trocar will allow for direct com-pression of the vessel. These two maneuvers should provide adequate hemostasis and time for a deliberate conversion to open with all members of the surgical team prepared and ready.

TAKE HOME POINTS

• The TAPP approach should be considered for patients with an indication for a preperitoneal repair (e.g. bilateral or recurrent inguinal or femo-ral hernia) in whom a TEPP approach is not feasible (e.g. due to obesity, previous pfannenstiel incision, or inadvertent peritoneal entry during access in an attempted TEPP repair).

• The right and left preperitoneal spaces should be dissected separately and 2 pieces of mesh used in cases of bilateral hernias to reduce the risk of recurrent hernia.

• Initial dissection in the preperitoneal space should remain close to the peritoneum to avoid inadver-tent injury to the femoral vessels.

• Adequate closure of the peritoneum after hernia repair is essential to prevent adhesions between bowel and mesh and to prevent internal herniation of bowel loops within the preperitoneal space.

SUGGESTED READINGS

Felix E. Causes of recurrence after laparoscopic hernioplasty. A multicenter study. Surg Endosc. 1998;12(3):226–231.

Lovisetto F. Laparoscopic transabdominal preperitoneal (TAPP) hernia repair: surgical phases and complications. Surg Endosc. 2007;21(4):646–652.

McCormack K. Laparoscopic techniques versus open tech-niques for inguinal hernia repair. Cochrane Database Syst Rev. 2003;(1):CD001785.

Rebuffat C. Laparoscopic repair of strangulated hernias. Surg Endosc. 2006;20(1):131–134.

Rosenberger RJ. The cutaneous nerves encountered during laparoscopic repair of inguinal hernia: new anatomical fi ndings for the surgeon. Surg Endosc. 2000;14(8):731–735.

Key Technical Steps

1. Incision of the peritoneum and development of the preperitoneal space.

2. Reduction of direct and/or femoral hernias medially. 3. Dissection of an indirect hernia sac off of the cord struc-

tures and subsequent reduction of the sac and the cord lipoma from within the deep inguinal ring.

4. Extensive peritoneal dissection with parietalization of the cord.

5. Placement of nonabsorbable mesh to cover the entire myopectineal orifi ce.

6. Closure of the peritoneum.

Potential Pitfalls

• Injury to femoral vessels from dissection in the “ triangle of doom” deep to the cord structures.

• Injury to genital branch of the genitofemoral nerve from injudicious use of cautery in the “triangle of pain” lateral to the cord structures.

• Traction injury to the cord structures during reduction of an indirect hernia if the sac is not adequately dissected off of the cord prior to reduction of the sac.

• Early recurrence if the peritoneum is not adequately dissected prior to mesh placement.

TABLE 1. Key Technical Steps and Potential Pitfalls

Dimick_Chap02.indd 9Dimick_Chap02.indd 9 3/15/2012 7:26:11 PM3/15/2012 7:26:11 PM

Page 44: Clinical Scenarios in Surgery - Booksca.ca...Clinical Scenarios in Surgery DECISION MAKING AND OPERATIVE TECHNIQUE Editors Justin B. Dimick, MD, MPH Assistant Professor of Surgery

10

Differential DiagnosisIn a patient with an intermittent groin bulge that is now fi xed, tender, and erythematous, complications of a groin hernia should be fi rst consideration in the differential diagnosis. However, there are several other possible etiologies to consider. Subcutaneous pathology, such as lipoma, groin abscess, or inguinal adenopathy, can present as a groin mass. Testicular pathology comprising torsion and epididymitis should also be considered, especially when the mass involves the scrotum. Vascular etiologies, such as aneurysmal or pseudoaneurysmal disease, should be considered in patients with a history of vascular disease and/or previous interventions at or near the femoral vessels.

Once the surgeon suspects groin hernia, it is impor-tant to discern inguinal from femoral hernia. To some degree, this can be ascertained on physical exam. For a femoral hernia, the bulge is below (and lateral) to the medial end of the inguinal ligament. In contrast, in an inguinal hernia, the bulge would be above the ingui-nal ligament (Figure 1). However, this distinction can be diffi cult to assess if the bulge is large, tender, and infl amed.

Most importantly, early identifi cation of complica-tions of groin hernia, such as incarceration or stran-gulation, is essential. Such complications change the time course of intervention. Incarcerated hernias can-not be reduced and therefore may progress to stran-gulation if they have not already. Strangulated hernia is by defi nition a hernia in which the blood supply of

the herniated viscus is compromised. For a reducible groin hernia, repair can be delayed and scheduled electively. But suspected incarceration and strangula-tion are surgical emergencies.

WorkupHistory and physical examination in patients with sus-pected incarcerated and/or inguinal hernia are often diagnostic. The decision to operate can often be made without further evaluation (Figure 2). Laboratory val-ues such as complete blood count, comprehensive metabolic panel, and lactate level can provide informa-tion about the patient’s hydration status and whether there is systemic infl ammatory response, which are important in assessing the likelihood of strangulation. However, these tests have a high sensitivity and low specifi city, that is, most patients with incarceration and strangulation will have normal or near-normal laboratory values. To avoid a high false-negative rate (i.e., missing the diagnosis when it is present), surgeons should err on the side of exploring patients when incar-ceration/strangulation are suspected. If there is sub-stantial uncertainty regarding the diagnosis, imaging studies can be obtained. If the patient is obstructed at the site of incarceration, plain fi lms of the abdomen will show signs of distended loops of bowel and air fl uid levels if the patient is obstructed (Figure 3). However, computed tomography (CT) imaging is the standard in emergency evaluation (Figure 4) if the clinical diag-nosis is in question after history, physical, and plain abdominal radiographs.

Presentation

A 61-year-old man presents to the emergency department with obstipation and left groin mass for 3 days. His past medical history was notable for chronic obstructive pulmonary disease, type II diabetes, obesity, hyperlipidemia, and schizophrenia. His surgical history was signifi cant for two prior inguinal hernia repairs on the left side. Due to his schizophrenia, he resides in an assisted living facility and comes in with a care-giver today. He describes an increase in abdominal pain and distention over the 3-day period. His oral intake has decreased, and he reports minimal urine output over the past 2 days. Physical exam is notable for a well-healed scar in the right lower quadrant at McBurney’s point and a large, 12- × 12-cm bulge in the left inguinal region. The mass is tender to palpation, erythematous, and nonreducible. Although the bulge has intermit-tently been present, both the patient and caregiver state that the size and tenderness are new in the past 2 days. Laboratory values were notable for a WBC of 8.7 and hematocrit of 42.4.

3 Incarcerated/Strangulated Inguinal HerniaMATTHEW W. RALLS and JUSTIN B. DIMICK

Dimick_Chap03.indd 10Dimick_Chap03.indd 10 3/16/2012 2:19:46 PM3/16/2012 2:19:46 PM

Page 45: Clinical Scenarios in Surgery - Booksca.ca...Clinical Scenarios in Surgery DECISION MAKING AND OPERATIVE TECHNIQUE Editors Justin B. Dimick, MD, MPH Assistant Professor of Surgery

Incarcerated/Strangulated Inguinal Hernia 11

DiscussionInguinal hernia repair is one of the most commonly performed surgical procedures worldwide. Over 800,000 inguinal hernia repairs are performed in the United States each year. Despite being a very com-mon operation, the relevant anatomy is complex and often diffi cult for students and surgical trainees to fully understand. An intimate knowledge of this anat-omy, however, is important, especially for addressing incarcerated or recurrent inguinal hernias. In these settings, the distortion of the tissues makes operative repair extremely challenging. In 1804, Astley Cooper

A BFIGURE 1 • Landmarks in discerning inguinal (A) versus femoral (B) hernia. (From Mulholland MW, et al. Greenfi eld’s Surgery: Scientifi c Principles & Practice. 4th ed. Philadelphia, PA: Lippincott Williams & Wilkins, 2006, with permission.)

FIGURE 2 • Erythema and swelling over left groin concern-ing for incarcerated hernia. This exam fi nding, coupled with appropriate presentation, is suffi cient cause for exploration.

FIGURE 3 • Plain fi lm of patient described in this clinical scenario. Distended loops of large bowel are concerning for a distal large bowel obstruction.

FIGURE 4 • CT showing left inguinal hernia.

stated, “No disease of the human body, belonging to the province of the surgeon, requires in its treatment a greater combination of accurate anatomic knowledge, with surgical skill, than hernia in all its varieties.”

Over the past two centuries, there have been many advances in groin hernia repair. The most frequently used technique in contemporary surgical practice is the tension-free mesh repair, or Lichtenstein repair. The laparoscopic totally extraperitoneal (TEP) is emerging as the most frequent minimally invasive approach and

Dimick_Chap03.indd 11Dimick_Chap03.indd 11 3/16/2012 2:19:47 PM3/16/2012 2:19:47 PM